Google Groups no longer supports new Usenet posts or subscriptions. Historical content remains viewable.
Dismiss

Ersetzen einer zweiwertigen Funktion durch nur eine nur einwertige Funktion?

40 views
Skip to first unread message

IV

unread,
Dec 29, 2016, 12:34:14 PM12/29/16
to
Hallo,

f1, f2, f seien einwertige komplexwertige Funktionen (oder reellwertige
Funktionen).
f1, f2 sind einstellige Funktionen.
f ist eine zweistellige Funktion.
f^-1 ist die Umkehrfunktion von f.
Die Argumente von f sind abhängig voneinander.
f: (f1(z),f2(z)) \mapsto f(f1(z),f2(z)) \in \mathbb{C} (or \mathbb{R})
f^-1: f(f1(z),f2(z)) \mapsto (f1(z),f2(z))
Wie kann man beweisen, daß die einstellige zweiwertige Funktion f^-1 nicht
durch nur eine einwertige Funktion oder durch nur eine Verkettung nur
einwertiger Funktionen ersetzt werden kann?
Kann jemand helfen?
Danke.

Jens Kallup

unread,
Dec 29, 2016, 1:12:57 PM12/29/16
to
Das hört sich an wie eine rekursive, die sich selbst wiederholenden
Funktion, die durch f1(z) und f2(z gegeben ist.
Wobei sich in der Funktion f zwei Schleifen befinden, die eigentlich
nur eine ist (Möbiosband???

f1(z) gegen f2(z) oder
f2(z) gegen f1(z)

wobei man nicht sagen kann, daß f1 = f2 oder f2 = f1 das gleiche sind.
Aber da ja alles in einen Gleichgewicht liegen muss/sollte, hier ein
C Quellcode Program, ungetestet:

//------%< snip ------
#include <stdio.h>

int z1 = -100; // untere Grenze
int z2 = 100; // obere Grenze

void f(int f1, int f2)
{
++f1;
--f2;

if ((f1 < z1) || (f2 < z1)) {
// Bedingung wahr: 100 = 100 ?
printf("ok\n");
return;
}

printf("Durchlauf: f1: %d, f2: %d\n",
f1,f2);

f(f1,f2);
}

int main()
{
f(z1,z2);
return 0;
}
// --->% snap ----

Hope this Helps
Jens

IV

unread,
Dec 29, 2016, 4:01:29 PM12/29/16
to
"Stefan Ram" schrieb im Newsbeitrag
news:Wertigkeit-2...@ram.dialup.fu-berlin.de...
>> Wie kann man beweisen, daß die einstellige zweiwertige Funktion f^-1
>> nicht durch nur eine einwertige Funktion oder durch nur eine Verkettung
>> nur einwertiger Funktionen ersetzt werden kann?
> Manchmal werden zweiwertige Funktionen als einwertige Funktion definiert,
> deren Wert ein Paar ist.
Um diese Interpretation auszuschließen, hatte ich mit angegeben: "f1, f2, f
seien einwertige komplexwertige Funktionen (oder reellwertige Funktionen)."

> Man nennt eine Funktion »n-wertig«, wenn sie ein n-Tupel ergibt. Dann ist
> es klar, daß eine Anwendung einer 2-wertigen Funktion nicht durch eine
> Anwendung einer 1-wertigen Funktion ersetzt werden kann, wenn der Wert
> des Terms gleich sein soll und man ein hinreichend feines Tupelsystem hat
> (bei dem ein 2-Tupel nicht als ein 1-Tupel angesehen werden kann, das ein
> 2-Tupel enthält).
Es scheint klar zu sein, aber wie kann man es mathematisch beweisen?

Torn Rumero DeBrak

unread,
Dec 29, 2016, 7:45:06 PM12/29/16
to
Am 29.12.2016 um 18:34 schrieb IV:
f^-1 ist eine zwei-wertige Funktion. Wie sollte dann aus einer
Verkettung von ein-wertigen Funktionen jemals etwas zwei-wertiges
werden?

Die Bezeichnung (f1(z),f2(z)) in

f: (f1(z),f2(z)) \mapsto f(f1(z),f2(z)) \in \mathbb{C} (or \mathbb{R})

zeigt doch, dass der Definitionsbereich für f nur ein-stellig ist
und nicht zwei-stellig und f eine Komposition ist:

f: z \mapsto (f1(z),f2(z)) \mapsto f(f1(z),f2(z)) \in \mathbb{C} (or
\mathbb{R})

oder was willst du mit der Schreibweise ausdrücken?



Jetzt einmal ein konkretes Beispiel:

Wähle

f1(z) = z
f2(z) = z^2
f(z1, z2) = 1/2 * (z1^2 + z2)

dann ist f(f1(z), f2(z)) = 1/2 * (z^2 + z^2) = z^2

f^-1: z^2 \mapsto (z, z^2)

Wie willst du jetzt f^-1 durch eine ein-wertige
Funktion oder durch eine Verkettung von
ein-wertigen Funktionen ersetzen, da f^-1 doch
offensichtlich zwei-wertig ist?

Kannst du mir sagen, wo ich verkehrt lieg, oder wo ich
dich nicht verstehe?

IV

unread,
Dec 30, 2016, 11:25:20 AM12/30/16
to
"Torn Rumero DeBrak" schrieb im Newsbeitrag
news:o44ame$cjr$1...@gioia.aioe.org...
>> f^-1: f(f1(z),f2(z)) \mapsto (f1(z),f2(z))
>> Wie kann man beweisen, daß die einstellige zweiwertige Funktion f^-1
>> nicht durch nur eine einwertige Funktion oder durch nur eine Verkettung
>> nur einwertiger Funktionen ersetzt werden kann?
> f^-1 ist eine zwei-wertige Funktion. Wie sollte dann aus einer Verkettung
> von ein-wertigen Funktionen jemals etwas zwei-wertiges werden?
> ...
> Kannst du mir sagen, wo ich verkehrt lieg, oder wo ich dich nicht
> verstehe?
In der Mathematik reicht es nicht, daß etwas klar und offensichtlich ist.
Alles muß bewiesen werden oder bewiesen worden sein.
Da ich kein Mathematiker bin, muß ich immer damit rechnen, daß meine
mathematischen Vermutungen falsch sein könnten.
Beispiel: Kann man eine mehrstellige Funktion durch einstellige Funktionen
ersetzen? Der Darstellungssatz/Superpositionssatz von Kolmogorov und Arnold
zeigt, daß jede mehrstellige stetige Funktion als Summer von Verkettungen
einstelliger Funktionen dargestellt werden kann.
Wie könnte man das was für Dich so offensichtlich ist beweisen?


Torn Rumero DeBrak

unread,
Dec 30, 2016, 12:47:27 PM12/30/16
to
Ein Beispiel bitte.
Und ein Link auf den Satz wäre schön, damit ich weiß,
wovon du sprichst.

> Wie könnte man das was für Dich so offensichtlich ist beweisen?
>
>

2 != 1, was muss man noch weiter beweisen?

Jens Kallup

unread,
Dec 30, 2016, 1:28:27 PM12/30/16
to
Am 30.12.2016 um 17:25 schrieb IV:
> "Torn Rumero DeBrak" schrieb im Newsbeitrag
> news:o44ame$cjr$1...@gioia.aioe.org...
>>> f^-1: f(f1(z),f2(z)) \mapsto (f1(z),f2(z))
>>> Wie kann man beweisen, daß die einstellige zweiwertige Funktion f^-1

es gbt die Bezeichnung "Operator" und "Funktion"
mehrere Funktionen - also zweiwertige können sein:

f2 (12, 2, 4, |N ... )
f1 (24, 4, 1, |N ... )

Wenn man sich der Mengenlehre annähern will, so ergeben
sich in Deinen Beispiel 2 nichtleere Mengen A und B
Falls dem Element aEA das Element bEB zugeordnet ist,
schreibt man f: a |--> b.

Zum Beispiel:

Menge A ( Schüler1, Schüler2, Schüler3 )
Menge B ( 1, 2, 3 )

Dann ist f: A -> B

wenn dann:

f: Schüler1 |--> 2
f: Schüler2 |--> 3
f: Schüler3 |--> 1

zugeordnet sind, dann wird die schreibweise

a^f = b

angewandt.
Man hat also 2f sowie 1n Möglichkeiten
f1 = Menge A
f2 = Menge B sowie die Verbindung zu beiden als f^-1

mit f^-1 wird nicht ausgesagt, das es hierbei um eine
Potenz handelt, sondern vielmehr die Umkehrung von
Funktionen

Die Umkehrfunktion f ist g

BeispielFrage: kann die Gleichung y = f(x) in die Form
x = g(y) gebracht werden?
(Hinterstübchen: f = g !)

f(x) = 2x - 1
y = 2x - 1
2x = y + 1
x = (y + 1) / 2

Die Umkehrfunktion von f lautet also f^-1(y) = (y + 1) / 2

Hope this helps
Jens

Jens Kallup

unread,
Dec 30, 2016, 1:39:51 PM12/30/16
to
> BeispielFrage: kann die Gleichung y = f(x) in die Form
> x = g(y) gebracht werden?
> (Hinterstübchen: f = g !)
>
> f(x) = 2x - 1
> y = 2x - 1
> 2x = y + 1
> x = (y + 1) / 2
>
> Die Umkehrfunktion von f lautet also f^-1(y) = (y + 1) / 2

Nachtrag als ZahlenBeispiel:
Um eine Funktion in einen Graphen zu zeichnen, benötigt man
2 Punkte x und y.

y sei 2
x sei 6

dann ergibt sich:

=> f^-1(2) = (5 + 1) / 2
=> (-1 + -2) = 6 / 2
=> -3 = 3
============================

IV

unread,
Dec 30, 2016, 4:32:47 PM12/30/16
to
"Torn Rumero DeBrak" schrieb im Newsbeitrag
news:o466jc$1f0u$1...@gioia.aioe.org...
>> In der Mathematik reicht es nicht, daß etwas klar und offensichtlich ist.
>> Alles muß bewiesen werden oder bewiesen worden sein.
>> Da ich kein Mathematiker bin, muß ich immer damit rechnen, daß meine
>> mathematischen Vermutungen falsch sein könnten.
>> Beispiel: Kann man eine mehrstellige Funktion durch einstellige
>> Funktionen ersetzen? Der Darstellungssatz/Superpositionssatz von
>> Kolmogorov und Arnold zeigt, daß jede mehrstellige stetige Funktion als
>> Summer von Verkettungen einstelliger Funktionen dargestellt werden kann.
> Ein Beispiel bitte.
> Und ein Link auf den Satz wäre schön, damit ich weiß, wovon du sprichst.
Das ist doch hier nicht der Punkt. Es ist nur so ein beliebiges Beispiel, wo
etwas ganz Offensichtliches (nämlich daß eine mehrstellige Funktion nicht
durch einstellige Funktionen ersetzt werden kann) falsch ist.
Wikipedia: Kolmogorov-Arnold representation theorem
https://en.wikipedia.org/wiki/Kolmogorov%E2%80%93Arnold_representation_theorem
Ich denke, die Umkehrung dieses Satzes bringt nichts für die Beantwortung
der eingangs gestellten Frage.

Torn Rumero DeBrak

unread,
Dec 30, 2016, 7:01:49 PM12/30/16
to
Der Satz sagt nicht über ein-stellige Funktionen, sondern über
zwei-stellige.

Zitat:
...can be represented as a superposition of continuous functions of two
variables.
Zitatende

Also zurück zur Ausgangsfrage und die Bitte um ein Beispiel.

PS:

Wie ist die Antwort zu meinem Beispiel aus einem anderen Post?
Zur Erinnerung:

Wähle

f1(z) = z
f2(z) = z^2
f(z1, z2) = 1/2 * (z1^2 + z2)

dann ist f(f1(z), f2(z)) = 1/2 * (z^2 + z^2) = z^2

f^-1: z^2 \mapsto (z, z^2)

Wie willst du jetzt f^-1 durch eine ein-wertige
Funktion oder durch eine Verkettung von
ein-wertigen Funktionen ersetzen, da f^-1 doch
offensichtlich zwei-wertig ist?

Kannst du mir sagen, wo ich verkehrt liege, oder wo ich
dich nicht verstehe?


Sind die gewählten Funktionen nicht passend ausgesucht?
Noch einfacher als lineráre und quadratische Funktionen geht
ja schon nicht mehr.

IV

unread,
Dec 31, 2016, 8:25:18 AM12/31/16
to
"Torn Rumero DeBrak" schrieb im Newsbeitrag
news:o46sh9$qcu$1...@gioia.aioe.org...
>>>>>> In der Mathematik reicht es nicht, daß etwas klar und offensichtlich
>>>>>> ist. Alles muß bewiesen werden oder bewiesen worden sein.
>>>>>> Da ich kein Mathematiker bin, muß ich immer damit rechnen, daß meine
>>>>>> mathematischen Vermutungen falsch sein könnten.
>>>> Beispiel: Kann man eine mehrstellige Funktion durch einstellige
>>>> Funktionen ersetzen? Der Darstellungssatz/Superpositionssatz von
>>>> Kolmogorov und Arnold zeigt, daß jede mehrstellige stetige Funktion als
>>>> Summe von Verkettungen einstelliger Funktionen dargestellt werden kann.
>>> Ein Beispiel bitte.
>>> Und ein Link auf den Satz wäre schön, damit ich weiß, wovon du sprichst.
>> Das ist doch hier nicht der Punkt. Es ist nur so ein beliebiges Beispiel,
>> wo etwas ganz Offensichtliches (nämlich daß eine mehrstellige Funktion
>> nicht durch einstellige Funktionen ersetzt werden kann) falsch ist.
>> Wikipedia: Kolmogorov-Arnold representation theorem
>> https://en.wikipedia.org/wiki/Kolmogorov%E2%80%93Arnold_representation_theorem
>> Ich denke, die Umkehrung dieses Satzes bringt nichts für die Beantwortung
>> der eingangs gestellten Frage.
> Der Satz sagt nicht über ein-stellige Funktionen, sondern über
> zwei-stellige.
> Zitat:
> ...can be represented as a superposition of continuous functions of two
> variables.
> Zitatende
Es gibt offenbar mehrere Varianten entsprechender Sätze. Gleich im 2. Absatz
des Wikipedia-Artikels: "The works of Kolmogorov and Arnold established that
if f is a multivariate continuous function, then f can be written as a
finite composition of continuous functions of a single variable and the
binary operation of addition." Und schau mal kurz auf die Formeln dort.
Im übrigen gilt der Satz nur für Funktionen im Reellen.

IV

unread,
Dec 31, 2016, 8:43:56 AM12/31/16
to
"Torn Rumero DeBrak" schrieb im Newsbeitrag
news:o46sh9$qcu$1...@gioia.aioe.org...
>>>> f^-1: f(f1(z),f2(z)) \mapsto (f1(z),f2(z))
>>>> Wie kann man beweisen, daß die einstellige zweiwertige Funktion f^-1
>>>> nicht durch nur eine einwertige Funktion oder durch nur eine Verkettung
>>>> nur einwertiger Funktionen ersetzt werden kann?
>>> In der Mathematik reicht es nicht, daß etwas klar und offensichtlich
>>> ist. Alles muß bewiesen werden oder bewiesen worden sein.
> Also zurück zur Ausgangsfrage und die Bitte um ein Beispiel.
> ...
> f^-1: z^2 \mapsto (z, z^2)
> Wie willst du jetzt f^-1 durch eine ein-wertige Funktion oder durch eine
> Verkettung von ein-wertigen Funktionen ersetzen, da f^-1 doch
> offensichtlich zwei-wertig ist?
> Kannst du mir sagen, wo ich verkehrt liege, oder wo ich dich nicht
> verstehe?
Die Frage ist doch, wie man beweisen kann, daß es nicht gehen kann.
Auch wenn es (für Dich) offensichtlich ist - wie kann man es beweisen?
Mithilfe der Mengenlehre/Relationen?
Wenn der Beweis hier gelingt, könnte ich damit einen wertvollen,
fruchtbringenden anderen neuen mathematischen Satz beweisen.


Torn Rumero DeBrak

unread,
Dec 31, 2016, 9:10:15 AM12/31/16
to
Oh, da scheint die Rücknahme des Postings nicht funktioniert zu haben.

> Es gibt offenbar mehrere Varianten entsprechender Sätze. Gleich im 2.
> Absatz des Wikipedia-Artikels: "The works of Kolmogorov and Arnold
> established that if f is a multivariate continuous function, then f can
> be written as a finite composition of continuous functions of a single
> variable and the binary operation of addition." Und schau mal kurz auf
> die Formeln dort.
> Im übrigen gilt der Satz nur für Funktionen im Reellen.
>

Und außerdem weiß man beim Lesen des Artikels nicht, warum einmal
"multivariable functions" und ein anderes Mal "multivariate functions"
gebraucht werden, oder ob diese Ausdrücke eine gleiche oder
verschiedene Bedeutung haben.

H0Iger SchuIz

unread,
Dec 31, 2016, 9:11:43 AM12/31/16
to
IV <ivgr...@onlinehome.de> wrote:

> "Torn Rumero DeBrak" schrieb im Newsbeitrag
> news:o46sh9$qcu$1...@gioia.aioe.org...
> >>>> f^-1: f(f1(z),f2(z)) \mapsto (f1(z),f2(z))
> >>>> Wie kann man beweisen, daß die einstellige zweiwertige Funktion f^-1
> >>>> nicht durch nur eine einwertige Funktion oder durch nur eine Verkettung
> >>>> nur einwertiger Funktionen ersetzt werden kann?
> >>> In der Mathematik reicht es nicht, daß etwas klar und offensichtlich
> >>> ist. Alles muß bewiesen werden oder bewiesen worden sein.
> > Also zurück zur Ausgangsfrage und die Bitte um ein Beispiel.
> > ...
> > f^-1: z^2 \mapsto (z, z^2)
> > Wie willst du jetzt f^-1 durch eine ein-wertige Funktion oder durch eine
> > Verkettung von ein-wertigen Funktionen ersetzen, da f^-1 doch
> > offensichtlich zwei-wertig ist?
> > Kannst du mir sagen, wo ich verkehrt liege, oder wo ich dich nicht
> > verstehe?
> Die Frage ist doch, wie man beweisen kann, daß es nicht gehen kann.
> Auch wenn es (für Dich) offensichtlich ist - wie kann man es beweisen?
> Mithilfe der Mengenlehre/Relationen?

Wie kann man beweisen, dass man eine reellwertige Funktion i.A. nicht
durch eine natürlichwertige "ersetzen" kann? Wenn die zu ersetzende
Funktion Funktionswerte liefert, die nicht im Wertebereich der
Ersatzfunktion vorkommen können, kann's nicht passen.

Es ist aber auch mir nicht klar, worauf das hinausläuft. Ein Beispiel
wäre in der Tat hilfreich. Aber dann müsste Nummer vier wohl wieder
eines ihrer Geheimnisse verraten ...

hs

IV

unread,
Dec 31, 2016, 10:21:26 AM12/31/16
to
"H0Iger SchuIz" schrieb im Newsbeitrag
news:1mz4mo0.p77y7n1dxdqcdN%q...@gmx.net...
>>>> f^-1: f(f1(z),f2(z)) \mapsto (f1(z),f2(z))
>>>> Wie kann man beweisen, daß die einstellige zweiwertige Funktion f^-1
>>>> nicht durch nur eine einwertige Funktion oder durch nur eine Verkettung
>>>> nur einwertiger Funktionen ersetzt werden kann?
> Es ist aber auch mir nicht klar, worauf das hinausläuft. Ein Beispiel wäre
> in der Tat hilfreich.
"The elementary functions are understood here to be those which are obtained
in a finite number of steps by performing algebraic operations and taking
exponentials and logarithms." [Ritt 1925]
Meine Interpretation: Die Elementaren Funktionen nach Ritt sind also genau
die (einstelligen einwertigen) Funktionen, die durch Verkettung einer
endlichen Anzahl von ein- oder mehrstelligen algebraischen Funktionen, exp
und/oder ln erzeugt werden. (Die Identitätsfunktion ist eine algebraische
Funktion).
Nach Ritts Satz [Ritt 1925] kann eine Elementare Funktion F aber nur eine
Elementare Funktion als Umkehrfunktion haben, wenn F als Verkettung einer
endlichen Anzahl von e i n s t e l l i g e n algebraischen Funktionen, exp
und/oder ln dargestellt werden kann.
Beispiel:
F(x) = xe^x
Die Problemstellung: Ist die Umkehrfunktion von F eine Elementare Funktion?
F(x) = A(x,e^x); A: die zweistellige algebraische Funktion (x1,x2) \mapsto
x1*x2, wobei x1 und x2 zwar abhängig voneinander, aber algebraisch
unabhängig voneinander sind.
Die Umkehrfunktion von A ist A^-1: A(x1,x2) \mapsto (x1,x2).
A^-1 ist also eine z w e i w e r t i g e Funktion, und deshalb laut
Definition keine Elementare Funktion.
Wie kann man beweisen, daß die einstellige zweiwertige Funktion A^-1 nicht
durch nur eine einwertige Funktion oder durch nur eine Verkettung nur
einwertiger Funktionen ersetzt werden kann?
Wenn man das beweist, kann man ganz schnell Ritts Satz beweisen - und ihn
auf allgemeine Funktionen in geschlossenen Ausdrücken erweitern.
Wir wissen ja, die Umkehrfunktion von F ist die Lambert-W-Funktion. Nach
Ritts Satz kann sie keine Elementare Funktion sein.


H0Iger SchuIz

unread,
Dec 31, 2016, 10:33:21 AM12/31/16
to
IV <ivgr...@onlinehome.de> wrote:

> A: die zweistellige algebraische Funktion (x1,x2) \mapsto
> x1*x2, wobei x1 und x2 zwar abhängig voneinander,

Keine Ahnung, was das meint. Inwiefern "abhängig"?

> aber algebraisch
> unabhängig voneinander sind.
> Die Umkehrfunktion von A ist A^-1: A(x1,x2) \mapsto (x1,x2).

Bijektive Funktionen haben Umkehrfunktionen. Diese Funktion $A$ ist aber
nicht injektiv, wie man an $A(1,2) = 1 \cdot 2 = 2 = 2 \cdot 1 = A(2,1)$
erkennt.

Dieses Beispiel hilft mir dann auch nicht.

hs

Jens Kallup

unread,
Dec 31, 2016, 10:59:01 AM12/31/16
to
Am 31.12.2016 um 16:21 schrieb IV:
> Nach Ritts Satz [Ritt 1925] kann eine Elementare Funktion F aber nur
> eine Elementare Funktion als Umkehrfunktion haben, wenn F als Verkettung
> einer endlichen Anzahl von e i n s t e l l i g e n algebraischen
> Funktionen, exp und/oder ln dargestellt werden kann.
> Beispiel:
> F(x) = xe^x
> Die Problemstellung: Ist die Umkehrfunktion von F eine Elementare Funktion?

Ihr wisst aber schon, das F^-1 nix mit Potenzen zu tun hat, das man dann
auch nicht so einfach mit ln spielereien machen kann.
^-1 bedeutet, das aus dem vorhandenen (ich sage mal jetzt - in Anbindung
an das elektronische Gebiet - aus zwei UND-Gatter, eines wird, und dann
negiert wird.

Also:
UND-Gatter 1: 01
UND-Gatter 2: 11
------------------
UND-Gatter 3: 11
==================

Wenn man nun "positiven" Strom in anlegt:

+ o-------> UND 1 >----+
|
o---| |R| Widerstand
|
- o-------< UND 2 <----+

dann ergibt sich aud der minus Seite ein ungleichgewicht.
Andersherum das selbe, nur dann auf der positiven Seite.

Und Ihr wisst ja, das alles im Gleichgewicht stehen sollte.
Und weil man im Haushalt aus der 220 Volt Steckdose einen
Gleichstrom erhält, kann man den Stecker des jeweiligen Gerätes
vertauschen, also _umkehren_ .

Da dies aber nicht gewünscht ist, weil rückwärtsfahren langsamer ist
als vorwärts, ist im Gerät ein zusätzliches Bauteil/Element eingebaut,
das dafür sorgt, das auch am Gerät _gleichstrom_ erhalten wird.

Ein weiteres Beispiel?
Gerne:
Im ComputerBereich gibt es das sogenannte Parityflag, welches dazu dient
einen Überlauf zu verhindern.
Wenn zum Beispiel auf die Finärfolge 11 ein weiteres Bit hinzugefügt
wird, weill vielleicht das Ergebnis so verlangt (etwa in dual:
100 * 10 = 1000, wird dann (grob gesagt: auf 111 ein Bit hinzugefügt:
1111 (was dann letzendlich auch eine größere Zahl ergibt)

Gut, diese Ausage ist nicht maßstabsgetreu und soll nur zur Darstellung
dienen.

Aber ich denke das reicht, und ist auch ein anderes Grimms Märchen.

Gruß
Jens

H0Iger SchuIz

unread,
Dec 31, 2016, 11:18:31 AM12/31/16
to
Jens Kallup <jka...@web.de> wrote:

> Am 31.12.2016 um 16:21 schrieb IV:
> > Nach Ritts Satz [Ritt 1925] kann eine Elementare Funktion F aber nur
> > eine Elementare Funktion als Umkehrfunktion haben, wenn F als Verkettung
> > einer endlichen Anzahl von e i n s t e l l i g e n algebraischen
> > Funktionen, exp und/oder ln dargestellt werden kann.
> > Beispiel:
> > F(x) = xe^x
> > Die Problemstellung: Ist die Umkehrfunktion von F eine Elementare Funktion?
>
> Ihr wisst aber schon, das F^-1 nix mit Potenzen zu tun hat, das man dann
> auch nicht so einfach mit ln spielereien machen kann.

Mach Sachen, na sowas. Aber mit dem wirren Zeug, das Herr Kallup
anführt, hat es etwas zu tun?

Na dann bin ich ja beruhigt.

hs

Jens Kallup

unread,
Dec 31, 2016, 11:27:14 AM12/31/16
to
Am 31.12.2016 um 17:18 schrieb H0Iger SchuIz:
> Ist die Umkehrfunktion von F eine Elementare Funktion

Hier habe ich vielleicht etwas besseres gefunden:

http://www.mathebibel.de/umkehrfunktion

IV

unread,
Dec 31, 2016, 11:40:10 AM12/31/16
to
"H0Iger SchuIz" schrieb im Newsbeitrag
news:1mz4r44.1t6lp971toutw5N%q...@gmx.net...
>> A: die zweistellige algebraische Funktion (x1,x2) \mapsto
>> x1*x2, wobei x1 und x2 zwar abhängig voneinander
> Keine Ahnung, was das meint. Inwiefern "abhängig"?
Keine Ahnung, ob es hier einen oder keinen Unterschied macht, daß x1 und x2
miteinander korreliert (durch eine Funktion miteinander verknüpft) sind.
Wenn x1 und x2 nicht miteinander korreliert sind, dann ist alles klar: Satz
von der Invarianz der Dimension - A kann nicht bijektiv sein, also keine
Umkehrfunktion haben; Ursache: verschiedene Mächtigkeit von Definitions- und
Ziel-/Bildmenge: Abbildung einer Ebene auf eine Kurve.
Nun ist aber x1 = x, x2 = e^x. x2 ist also eine (stetige) Funktion von x1.
Die Definitionsmenge (x,e^x) ist eine Kurve. Sie wird durch A auf eine Kurve
abgebildet. Kann die Funktion A auf einer mehrelementigen Definitionsmenge
bijektiv sein? Die Mächtigkeit und damit der Satz von der Invarianz der
Dimension greift ja hier nicht.
Ich hatte gehofft, die Bemerkung von Van der Waerden "Algebraisch
unabhängige Elemente kann man wie Unbestimmte behandeln, da ihre
algebraischen Eigenschaften dieselben sind." könnte hier weiterhelfen.

>> aber algebraisch unabhängig voneinander sind.
>> Die Umkehrfunktion von A ist A^-1: A(x1,x2) \mapsto (x1,x2).
> Bijektive Funktionen haben Umkehrfunktionen. Diese Funktion $A$ ist aber
> nicht injektiv, wie man an $A(1,2) = 1 \cdot 2 = 2 = 2 \cdot 1 = A(2,1)$
> erkennt.
> Dieses Beispiel hilft mir dann auch nicht.
Aber es gilt ja eben nicht nur A: (x1,x2) \mapsto x1*x2, sondern auch noch
x1=x, x=e^x - wo Ihr meintet, ich dürfe nicht definieren A: (x,e^x) \mapsto
x*e^x.
Wie kann man nun beweisen, daß die Funktion A^-1 nicht durch eine andere
elementare Funktion ersetzt werden kann?



IV

unread,
Dec 31, 2016, 11:47:36 AM12/31/16
to
"Jens Kallup" schrieb im Newsbeitrag news:o48kk4$us8$1...@news.albasani.net...
>>>> f^-1 ist die Umkehrfunktion von f.
>>>> ...
>>>> Wie kann man beweisen, daß die einstellige zweiwertige Funktion f^-1
>>>> nicht durch nur eine einwertige Funktion oder durch nur eine Verkettung
>>>> nur einwertiger Funktionen ersetzt werden kann?
> Ihr wisst aber schon, das F^-1 nix mit Potenzen zu tun hat, das man dann
> auch nicht so einfach mit ln-Spielereien machen kann.
> ^-1 bedeutet, dass aus dem vorhandenen (ich sage mal jetzt - in Anbindung
> an das elektronische Gebiet) aus zwei UND-Gattern ...
(Du hast wohl schon Silvestertrunk gehabt?) Ich denke mal, meine
mathematische Frage verlangt nach einer mathematischen Antwort - also nach
einer ohne Computerprogramm und Logikgattern, es sei denn, daß damit ein
mathematisch korrekter Beweis möglich ist.


Jens Kallup

unread,
Dec 31, 2016, 1:05:14 PM12/31/16
to
Am 31.12.2016 um 17:40 schrieb IV:

> Wie kann man nun beweisen, daß die Funktion A^-1 nicht durch eine andere
> elementare Funktion ersetzt werden kann?

vielleicht folgendes:

p = Polynom;

Nullstellen, Linearfaktoren
Def.: Zahl x_1 heißt Nullstelle, wenn gilt: p(x_1) = 0
Ist x_1 Nullstelle p(x) vom Grad n > 0, kann man den
Linearfaktor (x - x_1) ohne Rest dividieren:
p(x) = (x - x_1) * pn-1^(x)

Beispiel-Aufgabe:

p(x) = x^2 + x - 12


Lösen/ung Erklärung

x^2 + x - 12 = 0 | -12 negetiert auf andere Seite:
x^2 + x = 12 | quadratische Ergänzung auf beide Seiten:

x^2 + x + (1/2)^2 = (1/2)^2 + 12 | 1nen Bruch quadrieren:

x^2 + x + (1/2)^2 = (1^2 / 2^2) + 12 | ausrechnen
x^2 + x + (1/2)^2 = (1 / 4) + 12 | ...

x^2 + x + (1/2)^2 = (1 / 4) + ((12 * 4) / 4) | multiplizieren ...
x^2 + x + (1/2)^2 = (1 / 4) + (48 / 4) | da Bruch gleichnamig, add

x^2 + x + (1/2)^2 = 49/4 | -> (1 + 48) / 4


(x + 1/2)^2 = 49/4 | linke Seite siehe binomische Formeln

x + 1/2 = +- wurzel(49/4) | auf beiden Seiten Wurzeln:

x_1 + 0,5 = wurzel(49/4)
x_1 + 0,5 = wurzel(12,25)
x_1 + 0,5 = 3,5 | 0,5 abziehen auf beide Seiteb
x_1 = -0,5 + 3,5
x_1 = 3

x_2 + 0,5 = -(wurzel(49/4))
x_2 + 0,5 = -(wurzel(12,25))
x_2 + 0,5 = -3,5
x_2 = -0,5 - 3,5
x_2 = -4

Lösungsmengen: (3;4)


Beispiel 2: Polynomdivision

Linearfaktor: (x - 1)

(x^4 - x^3 + x^2 + 9x - 10 : (x-1) = x^3 + x - 10 (höchter Grad = x^4)

Lösung:
(x^4 - x^3 + x^2 + 9x - 10 | wir addieren x^3
x^4 + x^3 = x^3
------------------------- | x^4 kürzt sich weg
x^2 + 9x - 10 | -x wird negetiert
x^2 - x - 10 = x^3 + x
------------- | x^2 kürzt sich weg
-10x - 10
-10x - 10 = x^3 + x - 10 | x kürzt sich weg
---------
0 = x^3 + x - 10



Beispiel 3 - Horner-Schema:

(x^3 + x + 10) : (x + 2) = x^2 - 2x + 5
x^3 + 2x^2
----------
- 2x^2
- 2x^2 -4x
----------
5x
5x + 10
-------
0

Alternativ mit Horner-Schema:

| 1 0 1 10
+| -2 4 -10
-+----------------
| 1 -2 5 0

IV

unread,
Dec 31, 2016, 1:23:45 PM12/31/16
to
"Jens Kallup" schrieb im Newsbeitrag news:o48s0p$gta$1...@news.albasani.net...
>>> Beispiel:
>>> F(x) = xe^x
>>> Die Problemstellung: Ist die Umkehrfunktion von F eine Elementare
>>> Funktion?
>>> F(x) = A(x,e^x); A: die zweistellige algebraische Funktion (x1,x2)
>>> \mapsto x1*x2, wobei x1 und x2 zwar abhängig voneinander, aber
>>> algebraisch unabhängig voneinander sind.
>> Wie kann man nun beweisen, daß die Funktion A^-1 nicht durch eine andere
>> elementare Funktion ersetzt werden kann?
> vielleicht folgendes: ...
Nee, bitte nicht.

Es gibt unendlich viele Elementare Funktionen.



Jens Kallup

unread,
Dec 31, 2016, 1:26:39 PM12/31/16
to
Polynome kann man addieren, subtrahieren und multiplizieren,
oder oder oder ...
Man erhält dann wieder ein Polynom.

Die Polynomdivision funktioniert nicht immer ohne Rest !!!

Man nennt dann den Quotienten f(x) und g(x)

f(x) / g(x) ist eine rationale Funktion, deren Bereich sich
über ganz IR erstreckt, _ohne_ die Nullstellen von g(x).

Jens

H0Iger SchuIz

unread,
Dec 31, 2016, 2:11:36 PM12/31/16
to
IV <ivgr...@onlinehome.de> wrote:

> "H0Iger SchuIz" schrieb im Newsbeitrag
> news:1mz4r44.1t6lp971toutw5N%q...@gmx.net...
> >> A: die zweistellige algebraische Funktion (x1,x2) \mapsto
> >> x1*x2, wobei x1 und x2 zwar abhängig voneinander
> > Keine Ahnung, was das meint. Inwiefern "abhängig"?
> Keine Ahnung, ob es hier einen oder keinen Unterschied macht, daß x1 und x2
> miteinander korreliert (durch eine Funktion miteinander verknüpft) sind.

Abhängig, korreliert, wie auch immer Nummer vier das nennen mag, so
scheint mir das doch keine Eigenschaft der Funktion $A$ zu sein, sondern
erst durch die Verknüpfung zustande zu kommen.

Die Funktion $A: \mathbb{R}^2 \to \mathbb{R}, (x_1,x_2) \mapsto x_1
\cdot x_2$ ist jedenfalls nicht bijektiv.

Die Funktion $A: \{(x_1,x_2) \in \mathbb{R}^2 \mid x_2 = e^{x_1}\} \to
\mathbb{R}, (x_1,x_2) \mapsto x_1 \cdot x_2$

vielleicht schon. Wer hat Nummer vier eigentlich erklärt, sie könne bei
Fragen wie der nach der Bijektivität so wesentliche Angaben wie
Definitions- und Wertebereich weglassen.

> Wenn x1 und x2 nicht miteinander korreliert sind, dann ist alles klar: Satz
> von der Invarianz der Dimension - A kann nicht bijektiv sein, also keine
> Umkehrfunktion haben; Ursache: verschiedene Mächtigkeit von Definitions- und
> Ziel-/Bildmenge: Abbildung einer Ebene auf eine Kurve.
> Nun ist aber x1 = x, x2 = e^x.

Warum sollte das so sein? $x_1$ und $x_2$ tauchen nur als gebundene
Variablen innerhalb der Funktionsdefinition auf. Außerhalb kann man sie
gar nicht verwenden. Die "Korrelation" entsteht erst durch die
Verknüpfung mit zwei einstelligen Funktionen. Sie ist keine Eigenschaft
von $A$. S.o.


> x2 ist also eine (stetige) Funktion von x1.

Nein. $x_2$ bezeichnet ein Element des Definitionsbereiches von $A$,
also keine Funktion.

> Die Definitionsmenge (x,e^x) ist eine Kurve.

Das ist ein geeordnetes Paar. Zunächste ist das keine Menge. Wie man
eine Menge, die Elemente dieser Form enthält, aufschreibt, kann man
weiter oben sehen. Hier gilt mal wieder: Grundlagen pauken!

> Sie wird durch A auf eine Kurve
> abgebildet. Kann die Funktion A auf einer mehrelementigen Definitionsmenge
> bijektiv sein?

Ja, kann. Durchrechnen! Ich vermute, die Bijketivität von $x \mapsto
e^x$ "schlägt durch".

Wenn man aber die "Korrelation" $x_2 = e^{x_1}$, die noch zu
formalisieren wäre verwendet, braucht man auch keine Umkehrfunktion von
$A$. Wenn man $x_1$ kennt, kennt man ja auch $x_2$.

> Die Mächtigkeit und damit der Satz von der Invarianz der
> Dimension greift ja hier nicht.
> Ich hatte gehofft, die Bemerkung von Van der Waerden "Algebraisch
> unabhängige Elemente kann man wie Unbestimmte behandeln, da ihre
> algebraischen Eigenschaften dieselben sind." könnte hier weiterhelfen.

> Aber es gilt ja eben nicht nur A: (x1,x2) \mapsto x1*x2, sondern auch noch
> x1=x, x=e^x

Sicher nicht.

> - wo Ihr meintet, ich dürfe nicht definieren A: (x,e^x) \mapsto
> x*e^x.

Ja, schriebe ich so nicht hin.

> Wie kann man nun beweisen, daß die Funktion A^-1 nicht durch eine andere
> elementare Funktion ersetzt werden kann?

Zunächst muss es eine Funktion überhaupt geben, damit man sie ersetzen
kann. Man sollte sich also zunächst Gedanken über die Bijektivität
machen. S.o.

hs

H0Iger SchuIz

unread,
Dec 31, 2016, 2:11:36 PM12/31/16
to
Jens Kallup <jka...@web.de> wrote:

> Am 31.12.2016 um 17:18 schrieb H0Iger SchuIz:
> > Ist die Umkehrfunktion von F eine Elementare Funktion

Nein, das schrieb ich nicht. Zitieren kann er auch nicht.

> Hier habe ich vielleicht etwas besseres gefunden:
>
> http://www.mathebibel.de/umkehrfunktion
^^^^^

Ich weiß nicht, ob ich eine Web-Seite mit so einem blöden Namen
empföhle. Davon mal ab, weiß Nummer vier aber sicherlich was eine
Umkehrfunktion ist -- bei Herrn Kallup wäre ich mir da aber nicht so
sicher.

Vielleicht liest der da mal nach, bevor er anderen etwas empfiehlt.

hs

Jens Kallup

unread,
Dec 31, 2016, 2:51:23 PM12/31/16
to
Am 31.12.2016 um 20:11 schrieb H0Iger SchuIz:
> Vielleicht liest der da mal nach, bevor er anderen etwas empfiehlt.

Naja, da sind Grundlagen erklärt.
Und empfohlen habe ich auch nix (jedenfalls nicht ausdrücklich);
weshalb ich auch nur "vielleicht" geschrieben habe.

Aber egal, wir wollen mal nicht so kleinkarriert in das Neue Jahre
flitschen.
Kann ja nur besser werden.

Ich weiss ja, Ihr seid die Chefs hier oben.
Na und, dafür, ich hier unten -__-

Und ja, solltet Ihr mir helfen auf den richtigen Weg zu kommen, dann
könnte ich _vielleicht_ sagen, hey Ihr seid aber nen duftes Folk.

Gruß
Jens

P.S.: Nicht alles so ernst nehmen, was ich schreibe, vielles sind
Ideen, die ich so habe. Ob sie stimmen, das ist eine andere
Grimms Märchen Geschichte.

IV

unread,
Dec 31, 2016, 3:14:42 PM12/31/16
to
"H0Iger SchuIz" schrieb im Newsbeitrag
news:1mz4y1a.imhvu9a0h7xnN%q...@gmx.net...
> Wer hat Nummer vier eigentlich erklärt, sie könne bei Fragen wie der nach
> der Bijektivität so wesentliche Angaben wie Definitions- und Wertebereich
> weglassen.
[Ritt 1925]: "The elementary functions are understood here to be those which
are obtained in a finite number of steps by performing algebraic operations
and taking exponentials and logarithms."
Ritt kommt ganz ohne Angabe des Definitionsbereichs aus.

> Zunächst muss es eine Funktion überhaupt geben, damit man sie ersetzen
> kann. Man sollte sich also zunächst Gedanken über die Bijektivität machen.
> S. o.
Deinem Aufschrei folgend sehe ich folgende Fälle für die Funktionen A.
a) A: {(x1,x2) \in \mathbb{C}^2 | x2 = e^x1} --> \mathbb{C}, (x1,x2) \mapsto
x1*x2
b) x1 \in I1, x2 \in I2; I1, I2: Intervall \subseteq \mathbb{C}; A: {(x1,x2)
\in I1 x I2 | x2 = e^x1} --> \mathbb{C}, (x1,x2) \mapsto x1*x2
c) A: {(x1,x2) \in \mathbb{R}^2 | x2 = e^x1} --> \mathbb{R}, (x1,x2) \mapsto
x1*x2
d) x1 \in I1, x2 \in I2; I1, I2: Intervall \subseteq \mathbb{R}; A: {(x1,x2)
\in I1 x I2 | x2 = e^x1} --> \mathbb{R}, (x1,x2) \mapsto x1*x2
Ohne die Bedingung x2 = e^x1 könnte man den Brouwerschen Satz von der
Invarianz der Dimension heranziehen und damit beweisen, daß die einzelnen
Funktionen A nicht bijektiv sind.
Wie kann man aber die Bijektivität oder Nicht-Bijektivität beweisen, wo x2 =
e^x1?
Hat jemand hier Ahnung davon? Kann mir bitte jemand helfen?

>> Kann die Funktion A auf einer mehrelementigen Definitionsmenge bijektiv
>> sein?
> Ja, kann. Durchrechnen! Ich vermute, die Bijketivität von $x \mapsto e^x$
> "schlägt durch".
Mit H0Iger-Schulzschem Maßstab: Der Begriff "schlägt durch" muß hier noch
mathematisch korrekt definiert werden!

IV

unread,
Dec 31, 2016, 4:19:14 PM12/31/16
to
"H0Iger SchuIz" schrieb im Newsbeitrag
news:1mz4y1a.imhvu9a0h7xnN%q...@gmx.net...
>>>> Wie kann man beweisen, daß die einstellige zweiwertige Funktion f^-1
>>>> nicht durch nur eine einwertige Funktion oder durch nur eine Verkettung
>>>> nur einwertiger Funktionen ersetzt werden kann?
> Zunächst muss es eine Funktion überhaupt geben, damit man sie ersetzen
> kann. Man sollte sich also zunächst Gedanken über die Bijektivität machen.
> S.o.
Wir haben Funktionen A: {(x1,x2) \in \mathbb{C}^2 | x2 = e^x1} -->
\mathbb{C}, (x1,x2) \mapsto x1*x2.
Die Bijektivität braucht hier nicht betrachtet werden, weil sowieso von
bijektiven Funktionen ausgegangen wird.
Ist A bijektiv, dann muß es eine algebraische Funktion A^-1 geben mit A^-1:
\mathbb{C} --> {(x1,x2) \in \mathbb{C}^2 | x2 = e^x1}, x1*x2 \mapsto
(x1,x2).
Da diese Funktion zweiwertig ist, kann sie keine Elementare Funktion sein.
Wenn A nicht durch nur eine einwertige Funktion dargestellt werden kann,
dann hat die Funktion A keine Umkehrfunktion die eine Elementare Funktion
ist.
Die Frage bleibt also: "Wie kann man beweisen, daß die einstellige

Jens Kallup

unread,
Dec 31, 2016, 4:50:12 PM12/31/16
to
Am 31.12.2016 um 19:23 schrieb IV:

> Es gibt unendlich viele Elementare Funktionen.

Verhalten von Polynomen im Unendlichen:
Wenn Grad(f) > 0, dann gilt:

limes f(x) = positiv unendlich, falls a_n > 0
x --> oo

limes f(x) = negativ unendlich, falls a_n < 0
x --> oo


_sowie_:


limes f(x) = falls n gerade und a_n > 0
x --> -oo oder falls n _nicht_ gerade und a_n < 0

limes f(x) = falls n _nicht_ gerade und a_n > 0
x --> -oo oder falls n gerade und an < 0

-----------------------------------

Rationale Funktionen der Form:

a_0 + a_1x + ... + a_mx^n = p_m(x)
f(x) = ---------------------------- ----------
b_0 + b_1x + ... + b_mx^n = q_n(x)


ist eine _geborchene rationale_ Funkton.

p ist Zählerpolynom von f
q ist Nennerpolynom von f

Rationale Funktionen sind bis auf die Nullstellen des Nennerpolynoms
überall definiert.

IV

unread,
Dec 31, 2016, 5:34:19 PM12/31/16
to
"Jens Kallup" schrieb im Newsbeitrag news:o4996j$rtk$1...@news.albasani.net...
IV schrieb:
>>>>>>>> f1, f2, f seien einwertige komplexwertige Funktionen (oder
>>>>>>>> reellwertige Funktionen).
>>>>>>>> f1, f2 sind einstellige Funktionen.
>>>>>>>> f ist eine zweistellige Funktion.
>>>>>>>> f^-1 ist die Umkehrfunktion von f.
>>>>>>>> Die Argumente von f sind abhängig voneinander.
>>>>>>>> f: (f1(z),f2(z)) \mapsto f(f1(z),f2(z)) \in \mathbb{C} (or
>>>>>>>> \mathbb{R})

Jens Kallup

unread,
Dec 31, 2016, 5:37:26 PM12/31/16
to
Verkettung:

f1 : A -> B
f2 : B -> C

sind Abbildungen.

nehmen wir aus einen Töpfchen jeweils ein Element
und verknüpfen es mit einen Element eines folgenden
Töpfchens, dann schreibt man:

(f2 o f1)(x) = (f2(f1(x))

man spricht hier von Verkettung:

f2 verkettet mit f1

könnte man vielleicht als "Reihenschaltung" ansehen?
-> Ich nehm aus Topf 1 was raus, lege es in Topf 2,
und so weiter...

Frage mich jetzt, ob das was mit Atomzerfall oder
Abkühlung zu tun hat?

Ich nehme aus Topf 1 die Heizung raus, um in Topf2
meinen Popo zu wärmen, während Topf 1 schon friert.

H0Iger SchuIz

unread,
Dec 31, 2016, 5:45:51 PM12/31/16
to
Nummer vier <ivgr...@onlinehome.de> wrote:


> Mit H0Iger-Schulzschem Maßstab: Der Begriff "schlägt durch" muß hier noch
> mathematisch korrekt definiert werden!

Dann mache sie das, wenn sie meint. Dann hat sie aber nicht verstanden,
dass dieser Begriff überhaupt nicht als mathematischer Begriff verwendet
wurde.

Viel Spaß noch.

hs

Torn Rumero DeBrak

unread,
Dec 31, 2016, 7:16:08 PM12/31/16
to
Ja wie beweist du denn sonst die Bijektivität? Du mußt
Surjektivität und Injektivität zeigen, oder, falls du
die nicht-Bijektivität zeigen willst, mußt du ein Beispiel finden.

Nehme einmal im Fall d) den Wert y1 im Bildbereich von A.
Dann muß es ein eindeutiges Paar (x1, x2) geben mit x1*x2 = y1
und x2 = e^x1, d.h. die Gleichung y1/x1 = e^x1 muss eine eindeutige
Lösung haben. Das ist gleichbedeutend mit x1*e^x1 = y1
Nun ist die Lambertsche Funktion aber in [-1/e, 0) nicht injektiv,
d.h. falls y1 in diesem Intervall liegt, gibt es zwei Werte
x1,1 und x1,2 mit x1,1 * e^x1,1 = y1 und x1,2 * e^x1,2 = y1.

Damit erhälst du auch 2 Paare (x1,1 , x2,1) und (x1,2 , x2,2)
mit der gesuchten Eigenschaft und damit ist die Umkehrfunktion nicht
bijektiv



Carlo XYZ

unread,
Jan 1, 2017, 2:31:09 AM1/1/17
to
"IV" <ivgr...@onlinehome.de> wrote:

> Wie kann man aber die Bijektivität oder Nicht-Bijektivität beweisen, wo x2 =
> e^x1?
> Hat jemand hier Ahnung davon? Kann mir bitte jemand helfen?

Ist dir überhaupt klar, dass die Menge \{(x,e^x)|x\in C\} auf
C\times C weder eine surjektive noch eine injektive Relation ist,
prima facie also gar keine Umkehrfunktion besitzt? Vermutlich nicht,
wenn dir der Begriff der Riemannschen Fläche nichts sagt:
<o3tpbe$jk8$1...@news.albasani.net>
Übungsaufgaben: Wenn f(z)=e^z also keine umkehrbare Funktion ist,
wie verträgt sich das mit Ritts Satz, den du angeblich verstanden
hast? Ist e^z elementar? Algebraisch? Dito log(z)? Gibt es eine
Darstellung von e^z gemäß Ritts Satz, und wenn ja, wieso?

Carlo XYZ

unread,
Jan 1, 2017, 2:31:31 AM1/1/17
to
q...@gmx.net (H0Iger SchuIz) wrote:

> Ja, kann. Durchrechnen! Ich vermute, die Bijketivität von $x \mapsto
> e^x$ "schlägt durch".

Faszinierend! Was ist daran bijektiv? War x\in R^+ vorausgesetzt?

H0Iger SchuIz

unread,
Jan 1, 2017, 4:42:46 AM1/1/17
to
Nope. Es war zunächst keine Definitions- und Wertemenge angegeben. Der
Rest war ein Schnellschuss. Insgesamt war in dem wohl zu wenig
Bijektivität. Es war wohl kein gutes. Die Frage verstehe ich immer noch
nicht.

Ist mir aber auch egal. Die Fragestellerin macht mir eh nicht den
Anschein, sich helfen lassen zu wollen.

hs

Detlef Müller

unread,
Jan 1, 2017, 6:55:47 AM1/1/17
to
Am 29.12.2016 um 22:01 schrieb IV:
[...]
> Um diese Interpretation auszuschließen, hatte ich mit angegeben: "f1,
> f2, f seien einwertige komplexwertige Funktionen (oder reellwertige
> Funktionen)."

>> Man nennt eine Funktion »n-wertig«, wenn sie ein n-Tupel ergibt. Dann
>> ist es klar, daß eine Anwendung einer 2-wertigen Funktion nicht durch
>> eine Anwendung einer 1-wertigen Funktion ersetzt werden kann, wenn
>> der Wert des Terms gleich sein soll und man ein hinreichend feines
>> Tupelsystem hat (bei dem ein 2-Tupel nicht als ein 1-Tupel angesehen
>> werden kann, das ein 2-Tupel enthält).
> Es scheint klar zu sein, aber wie kann man es mathematisch beweisen?

Mit f: C --> C und g: C --> C^2 ist
f(0) \in C,
aber
g(0) \in C^2.
Da die Anzahl der Komponenten von f(0) gleich 1 ist, die
Anzahl der Komponenten von g(0) hingegen 2, haben f(0) und
g(0) unterschiedliche Eigenschaften.
Daher gilt f(0) <> g(0).

Gleichheit von Abbildungen
bedeutet erstens Übereinstimmung der Definitionsbereiche
D_f=D_g=D sowie zweitens f(x)=g(x) für alle x aus D.

Insbesondere folgt aus f(0) <> g(0) schon f<>g.

Aber Stefans Erklärung ist doch viel klarer und meine
"klein-klein"-Ausführung dürfte hier eher verwirren
als klären.

Für alles, was über "f <> g, da die im(f) aus Skalaren,
im(g) hingegen aus 2-Tupeln Komplexer Zahlen besteht."
hinaus geht, wirst Du imo von Kennern belächelt und
lenkst Deine Zielgruppe vom Wesentlichen ab.

Gruß,
Detlef

--
Dr. Detlef Müller,
http://www.mathe-doktor.de oder http://mathe-doktor.de

IV

unread,
Jan 1, 2017, 7:30:33 AM1/1/17
to
"Jens Kallup" schrieb im Newsbeitrag news:o49bv6$g8g$1...@news.albasani.net...
> f2 verkettet mit f1
> könnte man vielleicht als "Reihenschaltung" ansehen?
Von Logikschaltungen zum Analogrechner.

> -> Ich nehm aus Topf 1 was raus, lege es in Topf 2, und so weiter...
> Frage mich jetzt, ob das was mit Atomzerfall oder Abkühlung zu tun hat?
Auch das Topfpotential der Schrödingergleichung will bedacht sein.

Hallo Jens,
===> Bitte eröffne doch für Deine "Thema-Verfehlt-Kommentare" (Neudeutsch:
off-topic) eigene Threads!

Hier geht es um Folgendes:
IV schrieb:
>>>>>>>> f1, f2, f seien einwertige komplexwertige Funktionen (oder
>>>>>>>> reellwertige Funktionen).
>>>>>>>> f1, f2 sind einstellige Funktionen.
>>>>>>>> f ist eine zweistellige Funktion.
>>>>>>>> f^-1 ist die Umkehrfunktion von f.
>>>>>>>> Die Argumente von f sind abhängig voneinander.
>>>>>>>> f: (f1(z),f2(z)) \mapsto f(f1(z),f2(z)) \in \mathbb{C} (or
>>>>>>>> \mathbb{R})

Carlo XYZ

unread,
Jan 1, 2017, 7:37:09 AM1/1/17
to
Detlef Müller <lef...@arcor.de> wrote:

> Gleichheit von Abbildungen
> bedeutet erstens Übereinstimmung der Definitionsbereiche
> D_f=D_g=D sowie zweitens f(x)=g(x) für alle x aus D.

Demnach wären

f:{1}->{2}, 1\mapsto 2 und g:{1}->{2,3}, 1\mapsto 2

gleich. Aber f ist surjektiv, g nicht. Seltsam:)

H0Iger SchuIz

unread,
Jan 1, 2017, 7:47:30 AM1/1/17
to
Jens Kallup <jka...@web.de> wrote:

> Nicht alles so ernst nehmen, was ich schreibe,

Keine Sorge, das passiert schon nicht.

Trotzdem nerven deine Beiträge so langsam. Womöglich kann man in
vorliegenden Thread auch anmerken, dass es ohnehin egal sei, weil man
immer noch nicht weiß, worum es eigentlich gehen soll.

hs


Carlo XYZ

unread,
Jan 1, 2017, 7:48:57 AM1/1/17
to
"IV" <ivgr...@onlinehome.de> wrote:

> Hallo Jens,
> ===> Bitte eröffne doch für Deine "Thema-Verfehlt-Kommentare" (Neudeutsch:
> off-topic) eigene Threads!

Och nö, das würde den Auflockerungseffekt mindern.

Du musst ja nicht alles genau studieren! Und überhaupt: manche Leute
sind glücklich, wenn "ihre" Threads besonders lang werden..

IV

unread,
Jan 1, 2017, 7:57:40 AM1/1/17
to
"H0Iger SchuIz" schrieb im Newsbeitrag
news:1mz65gh.1ehejlx1mciwzuN%q...@gmx.net...
> Die Fragestellerin macht mir eh nicht den Anschein, sich helfen lassen zu
> wollen.
Man kann das auch teamorientiert / lösungsorientiert formulieren. (Das
scheint Dir aber nicht zu liegen.)
Meint: Wenn einer auf eine Aufforderung nicht oder nicht wie gewünscht
reagiert, dann sollte man ihm den Sachverhalt eben mit etwas anderen,
leichter verständlichen Worten darlegen. (Ich kann doch nur auf das
reagieren, was ich halbwegs verstehe, und nur so antworten, wie ich es
verstehe.)
(Zu sagen: "Studiere erstmal Mathematik, erst dann kann ich mich mit Dir
unterhalten." ist nicht zielführend im Gespräch zwischen Mathematikern und
Nicht-Mathematikern.)
Antworten bitte in einer anderen Diskussion!

IV

unread,
Jan 1, 2017, 7:58:06 AM1/1/17
to
"H0Iger SchuIz" schrieb im Newsbeitrag
news:1mz65gh.1ehejlx1mciwzuN%q...@gmx.net...
> Die Fragestellerin macht mir eh nicht den Anschein, sich helfen lassen zu
> wollen.
Es geht hier doch um die Elementaren Funktionen oder Funktionen im
Allgemeinen. Es dürfte ersichtlich geworden sein, daß es um Funktionen im
Reellen oder im Komplexen geht. Um die Allgemeingültigkeit beizubehalten,
wollte ich die Definitionsmengen vorerst nicht weiter einschränken.
Meine Frage: Wird die Allgemeinheit eingeschränkt, wenn ich schreibe, die
Definitionsmenge X ist eine Teilmenge von \mathbb{R} oder von\mathbb{C}?

> Die Fragestellerin macht mir eh nicht den Anschein, sich helfen lassen zu
> wollen.
Die "Ablenkungen" mit konkreten Definitionsbereichen und Bijektivität werden
doch für die Lösung des mathematischen Problems gar nicht gebraucht. Oder
sehe ich das falsch?

> Die Fragestellerin macht mir eh nicht den Anschein, sich helfen lassen zu
> wollen.
>>> Wir haben Funktionen A: {(x1,x2) \in \mathbb{C}^2 | x2 = e^x1} -->
>>> \mathbb{C}, (x1,x2) \mapsto x1*x2.
>>> Die Bijektivität braucht hier nicht betrachtet werden, weil sowieso von
>>> bijektiven Funktionen ausgegangen wird.
>>> Ist A aber bijektiv, dann muß es eine algebraische Funktion A^-1 geben
>>> mit A^-1: \mathbb{C} --> {(x1,x2) \in \mathbb{C}^2 | x2 = e^x1}, x1*x2
>>> \mapsto (x1,x2).
>>> Da diese Funktion zweiwertig ist, kann sie keine Elementare Funktion
>>> sein.
>>> Wenn A nicht durch nur eine einwertige Funktion dargestellt werden kann,
>>> dann hat die Funktion A keine Umkehrfunktion die eine Elementare
>>> Funktion ist.
>>> Die Frage bleibt also: "Wie kann man beweisen, daß die einstellige

IV

unread,
Jan 1, 2017, 7:59:54 AM1/1/17
to
"Carlo XYZ" schrieb im Newsbeitrag
news:carloxyz-9181CF...@88-209-239-213.giganet.hu...
> Und überhaupt: manche Leute sind glücklich, wenn "ihre" Threads besonders
> lang werden...
Ich nicht!!! Ich hatte gehofft, sachliche Diskussionen wären möglich.

IV

unread,
Jan 1, 2017, 8:20:10 AM1/1/17
to
"H0Iger SchuIz" schrieb im Newsbeitrag
news:1mz65gh.1ehejlx1mciwzuN%q...@gmx.net...
>>> Womöglich kann man in vorliegenden Thread auch anmerken, dass es ohnehin
>>> egal sei, weil man immer noch nicht weiß, worum es eigentlich gehen
>>> soll.
Die Frage ist:
Ich habe beobachtet, daß die Elementaren Funktionen (nach Ritt) (das hier
ist nicht die Definition) einstellige einwertige Funktionen im Reellen oder
im Komplexen sind, die Verkettungen ein- oder m e h r stelliger einwertiger
Funktionen sind.
Elementare Funktionen, die eine elementare Funktion als Umkehrfunktion
haben, sind aber nur solche Verkettungen e i n s t e l l i g e r
Funktionen.
Wodurch werden alle mehrstellige Funktionen enthaltenden Verkettungen
ausgeschlossen?
Ich konnte das darauf zurückführen, daß eine mehrstellige Funktion, wenn sie
eine Umkehrfunktion hat, eine mehrwertige Funktion als Umkehrfunktion hat.
Eine mehrwertige Funktion, kann, wenn sie sich nicht durch eine einwertige
Funktion ersetzen läßt, aber keine Elementare Funktion sein, da diese laut
Definition nur einwertige Funktionen sind.
Die Frage bleibt also:
Die Frage bleibt also: "Wie kann man beweisen, daß eine einstellige
zweiwertige Funktion, deren Argumente einstellige einwertige Funktionen ein
und derselben Variablen x sind, nicht durch nur eine einwertige Funktion
oder durch nur eine Verkettung nur einwertiger Funktionen ersetzt werden
kann?"
Wenn nötig können die Funktionen lediglich im Reellen oder lediglich im
Komplexen betrachtet werden.



H0Iger SchuIz

unread,
Jan 1, 2017, 8:23:28 AM1/1/17
to
IV <ivgr...@onlinehome.de> wrote:

> "H0Iger SchuIz" schrieb im Newsbeitrag
> news:1mz65gh.1ehejlx1mciwzuN%q...@gmx.net...
> > Die Fragestellerin macht mir eh nicht den Anschein, sich helfen lassen zu
> > wollen.
> Man kann das auch teamorientiert / lösungsorientiert formulieren. (Das
> scheint Dir aber nicht zu liegen.)

Womöglich. Ich hab keine Ahnung, wie eine lösungsorientierte
Formulierung der Tatsache, dass mir das (wiederholte) Gemecker der
Fragestellerin auf den Keks geht. Die einzige Lösung, die mir da
einfällt, wäre, dass diejenige, die von den anderen etwas möchte, sich
gelegentlich etwas zurücknimmt.

> (Zu sagen: "Studiere erstmal Mathematik, erst dann kann ich mich mit Dir
> unterhalten."

Hat das so jemand gesagt? Oder wurde das Verbessern der Kenntnisse der
Grundlagen empfohlen.

> ist nicht zielführend im Gespräch zwischen Mathematikern und
> Nicht-Mathematikern.)

Wie zielführend sind denn Gespräche zwischen Mathematikern und
Nicht-Mathematikern _über_ _Mathematik_? Daran, sich mit Mathematik zu
beschäftigen, wenn man sich mit Mathematik beschäftigen möchte, kommt
man aber so oder so nicht vorbei.

hs

H0Iger SchuIz

unread,
Jan 1, 2017, 8:23:28 AM1/1/17
to
IV <ivgr...@onlinehome.de> wrote:

> "H0Iger SchuIz" schrieb im Newsbeitrag
> news:1mz65gh.1ehejlx1mciwzuN%q...@gmx.net...
> > Die Fragestellerin macht mir eh nicht den Anschein, sich helfen lassen zu
> > wollen.
> Es geht hier doch um die Elementaren Funktionen oder Funktionen im
> Allgemeinen. Es dürfte ersichtlich geworden sein, daß es um Funktionen im
> Reellen oder im Komplexen geht.

Wobei reell oder komplex erhebliche Unterschiede ausmachen kann.

> Um die Allgemeingültigkeit beizubehalten,
> wollte ich die Definitionsmengen vorerst nicht weiter einschränken.
> Meine Frage: Wird die Allgemeinheit eingeschränkt, wenn ich schreibe, die
> Definitionsmenge X ist eine Teilmenge von \mathbb{R} oder von\mathbb{C}?

> > Die Fragestellerin macht mir eh nicht den Anschein, sich helfen lassen zu
> > wollen.
> Die "Ablenkungen" mit konkreten Definitionsbereichen und Bijektivität werden
> doch für die Lösung des mathematischen Problems gar nicht gebraucht.

Was ist denn "das mathematische Problem"?

> Oder
> sehe ich das falsch?

Ja. Konkret: 1. Wenn man eine Umkehrfunktion konstatiert, sollte man
zunächst sicherstellen, dass die umzukehrende Funktion überhaupt eine
Umkehrfunktion besitzt.

2. Wenn man sich mit Fragen wie Mehrstelligkeit und Mehrwertigkeit
beschäftigt, dreht es sich sehr detailliert um Definitions- und
Wertebereich. Da wäre ich dann genau.

> > Die Fragestellerin macht mir eh nicht den Anschein, sich helfen lassen zu
> > wollen.
> >>> Wir haben Funktionen A: {(x1,x2) \in \mathbb{C}^2 | x2 = e^x1} -->
> >>> \mathbb{C}, (x1,x2) \mapsto x1*x2.
> >>> Die Bijektivität braucht hier nicht betrachtet werden, weil sowieso von
> >>> bijektiven Funktionen ausgegangen wird.

Wir haben die Zahlen -7. Positivität braucht nicht betrachtet zu werden,
weil sowieso von positiven Zahlen ausgegangen wird.

> >>> Ist A aber bijektiv,

Ist -7 aber positiv ...

Diese Ansage war gebührenfrei.

hs

H0Iger SchuIz

unread,
Jan 1, 2017, 8:27:06 AM1/1/17
to
IV <ivgr...@onlinehome.de> wrote:

> "H0Iger SchuIz" schrieb im Newsbeitrag
> news:1mz65gh.1ehejlx1mciwzuN%q...@gmx.net...
> >>> Womöglich kann man in vorliegenden Thread auch anmerken, dass es ohnehin
> >>> egal sei, weil man immer noch nicht weiß, worum es eigentlich gehen
> >>> soll.

IV <ivgr...@onlinehome.de> wrote:

> Die Frage bleibt also: "Wie kann man beweisen, daß eine einstellige
> zweiwertige Funktion, deren Argumente einstellige einwertige Funktionen ein
> und derselben Variablen x sind, nicht durch nur eine einwertige Funktion
> oder durch nur eine Verkettung nur einwertiger Funktionen ersetzt werden
> kann?"

Das wurde schon mehrfach beantwortet. Wenn das die Frage war, sollte der
Thread als "gelöst" markiert werden.

hs

IV

unread,
Jan 1, 2017, 8:39:00 AM1/1/17
to
"H0Iger SchuIz" schrieb im Newsbeitrag
news:1mz6fk9.2tk6641lt0zu7N%q...@gmx.net...
> >>> Die Bijektivität braucht hier nicht betrachtet werden, weil sowieso
> >>> von bijektiven Funktionen ausgegangen wird.
> Wir haben die Zahlen -7. Positivität braucht nicht betrachtet zu werden,
> weil sowieso von positiven Zahlen ausgegangen wird.
Tut mir leid, aber ich muß schon wieder "meckern":
Ist nicht das folgende Beispiel eher zutreffend:
Wir haben die Zahlen -7 (bijektiven Funktionen A). Positivität
(Nicht-Bijektivität) braucht nicht betrachtet zu werden, weil sowieso von n
e g a t i v e n Zahlen (bijektiven Funktionen) ausgegangen wird.

> >>> Ist A aber bijektiv,
> Ist -7 aber positiv ...
Ist -7 aber negativ ...


H0Iger SchuIz

unread,
Jan 1, 2017, 8:41:55 AM1/1/17
to
IV <ivgr...@onlinehome.de> wrote:

> "H0Iger SchuIz" schrieb im Newsbeitrag
> news:1mz6fk9.2tk6641lt0zu7N%q...@gmx.net...
> > >>> Die Bijektivität braucht hier nicht betrachtet werden, weil sowieso
> > >>> von bijektiven Funktionen ausgegangen wird.
> > Wir haben die Zahlen -7. Positivität braucht nicht betrachtet zu werden,
> > weil sowieso von positiven Zahlen ausgegangen wird.
> Tut mir leid, aber ich muß schon wieder "meckern":
> Ist nicht das folgende Beispiel eher zutreffend:

Nein.

IV

unread,
Jan 1, 2017, 8:46:08 AM1/1/17
to
"H0Iger SchuIz" schrieb im Newsbeitrag
news:1mz6g2r.1t9uid0qi67diN%q...@gmx.net...
>> Die Frage bleibt also: "Wie kann man beweisen, daß eine einstellige
>> zweiwertige Funktion, deren Argumente einstellige einwertige Funktionen
>> ein und derselben Variablen x sind, nicht durch nur eine einwertige
>> Funktion oder durch nur eine Verkettung nur einwertiger Funktionen
>> ersetzt werden kann?"
> Das wurde schon mehrfach beantwortet. Wenn das die Frage war, sollte der
> Thread als "gelöst" markiert werden.
Tut mir leid, ich habe das nicht bemerkt. Detlefs Antwort habe ich noch
nicht ausgewertet.
Ich habe bisher nur verstanden, es sei ganz offensichtlich, daß etwas
Zweiwertiges nicht durch eine einwertige Funktion ersetzt werden kann. Das
ist für mich aber doch kein mathematischer Beweis.
Kannst Du mir bitte den/die Posts nennen, in dem/denen das mathematische
Problem beantwortet wurde? Oder kannst Du mir die Antwort ganz kurz
formulieren?

H0Iger SchuIz

unread,
Jan 1, 2017, 8:55:40 AM1/1/17
to
IV <ivgr...@onlinehome.de> wrote:

> Ich habe bisher nur verstanden, es sei ganz offensichtlich, daß etwas
> Zweiwertiges nicht durch eine einwertige Funktion ersetzt werden kann.

Eben.

> Das
> ist für mich aber doch kein mathematischer Beweis.

Was ist an "offensichtlich" noch unklar?

> Kannst Du mir bitte den/die Posts nennen, in dem/denen das mathematische
> Problem beantwortet wurde? Oder kannst Du mir die Antwort ganz kurz
> formulieren?

Keine Lust.

hs

IV

unread,
Jan 1, 2017, 9:00:43 AM1/1/17
to
"H0Iger SchuIz" schrieb im Newsbeitrag
news:1mz6gq9.gcb63j1m3jbjcN%q...@gmx.net...
>>>>> Die Bijektivität braucht hier nicht betrachtet werden, weil sowieso
>>>>> von bijektiven Funktionen ausgegangen wird.
>>> Wir haben die Zahlen -7. Positivität braucht nicht betrachtet zu werden,
>>> weil sowieso von positiven Zahlen ausgegangen wird.
>> Tut mir leid, aber ich muß schon wieder "meckern":
>> Ist nicht das folgende Beispiel eher zutreffend:
>> Wir haben die Zahlen -7 (bijektiven Funktionen A). Positivität
>> (Nicht-Bijektivität) braucht nicht betrachtet zu werden, weil sowieso von
>> n e g a t i v e n Zahlen (bijektiven Funktionen) ausgegangen wird.
> Nein.
(Nun muß ich doch erst 5 Jahre Mathematik studieren, um die Antwort
nachvollziehen zu können.)

Vielleicht ist es nicht deutlich geworden:
Mir scheint, es geht doch wirklich nur um die bijektiven Funktionen. Ritts
Satz beginnt ja folgendermaßen: "If F(z) and its inverse are both
elementary, there exist n functions ...". Daß die Funktionen, um die es im
Satz geht, bijektiv sind, ist Teil der Voraussetzungen im Satz.
Meine Idee/Beobachtung war ja gerade, daß es nur darauf ankommt, zu zeigen
"daß eine einstellige zweiwertige Funktion, deren Argumente einstellige
einwertige Funktionen ein und derselben Variablen x sind, nicht durch nur
eine einwertige Funktion oder durch nur eine Verkettung nur einwertiger
Funktionen ersetzt werden kann".
Oder liege ich hier auch wieder falsch?
Es wäre schön, wenn Ihr mir kurz sagen könntet, w o ich hier falsch liege.

H0Iger SchuIz

unread,
Jan 1, 2017, 9:06:16 AM1/1/17
to
IV <ivgr...@onlinehome.de> wrote:

> "H0Iger SchuIz" schrieb im Newsbeitrag
> news:1mz6gq9.gcb63j1m3jbjcN%q...@gmx.net...
> >>>>> Die Bijektivität braucht hier nicht betrachtet werden, weil sowieso
> >>>>> von bijektiven Funktionen ausgegangen wird.
> >>> Wir haben die Zahlen -7. Positivität braucht nicht betrachtet zu werden,
> >>> weil sowieso von positiven Zahlen ausgegangen wird.
> >> Tut mir leid, aber ich muß schon wieder "meckern":
> >> Ist nicht das folgende Beispiel eher zutreffend:
> >> Wir haben die Zahlen -7 (bijektiven Funktionen A). Positivität
> >> (Nicht-Bijektivität) braucht nicht betrachtet zu werden, weil sowieso von
> >> n e g a t i v e n Zahlen (bijektiven Funktionen) ausgegangen wird.
> > Nein.
> (Nun muß ich doch erst 5 Jahre Mathematik studieren, um die Antwort
> nachvollziehen zu können.)

Nein.

> Vielleicht ist es nicht deutlich geworden:
> Mir scheint,

Soso.

> es geht doch wirklich nur um die bijektiven Funktionen.

Dann sollte sie auch Beispiele mit bijektiven Funktionen wählen.
Generell kann man Sätze nur dann anwenden, wenn die Voraussetzungen
erfüllt sind. Das muss man gegebenenfalls prüfen.

Auch diese Ansage war gebührenfrei.

hs

IV

unread,
Jan 1, 2017, 9:08:37 AM1/1/17
to
"H0Iger SchuIz" schrieb im Newsbeitrag
news:1mz6hdx.v36t6j12nbs22N%q...@gmx.net...
>> Ich habe bisher nur verstanden, es sei ganz offensichtlich, daß etwas
>> Zweiwertiges nicht durch eine einwertige Funktion ersetzt werden kann.
>> Das ist für mich aber doch kein mathematischer Beweis.
> Was ist an "offensichtlich" noch unklar?
Für mich als Nicht-Mathematiker: Alles!
Ich kann in meiner Veröffentlichung in einer mathematischen Fachzeitschrift
also schreiben, daß keine n-wertige Funktion durch nur eine m-wertige
Funktion (m < n) ersetzt werden kann / auf eine solche reduziert(?) werden
kann, ja?
Ein Beweis ist nicht nötig? (Manchmal hat die Mathematik ja auch geirrt, und
dann waren ganz offensichtliche Dinge plötzlich doch nicht wahr!)
Ein Beweis wäre mir aber sicherer. (Ich habe Detlefs Antwort noch nicht
ausgewertet.)


H0Iger SchuIz

unread,
Jan 1, 2017, 9:12:15 AM1/1/17
to
IV <ivgr...@onlinehome.de> wrote:

> "H0Iger SchuIz" schrieb im Newsbeitrag
> news:1mz6hdx.v36t6j12nbs22N%q...@gmx.net...
> >> Ich habe bisher nur verstanden, es sei ganz offensichtlich, daß etwas
> >> Zweiwertiges nicht durch eine einwertige Funktion ersetzt werden kann.
> >> Das ist für mich aber doch kein mathematischer Beweis.
> > Was ist an "offensichtlich" noch unklar?
> Für mich als Nicht-Mathematiker: Alles!
> Ich kann in meiner Veröffentlichung in einer mathematischen Fachzeitschrift
> also schreiben,

Hier wäre wohl der Konjunktiv angemessen, oder?

> daß keine n-wertige Funktion durch nur eine m-wertige
> Funktion (m < n) ersetzt werden kann / auf eine solche reduziert(?) werden
> kann, ja?
> Ein Beweis ist nicht nötig?

Das klärte ich mit den Gutachtern/ bzw. der Redaktion.


> (Manchmal hat die Mathematik ja auch geirrt, und
> dann waren ganz offensichtliche Dinge plötzlich doch nicht wahr!)

Soso.

> Ein Beweis wäre mir aber sicherer. (Ich habe Detlefs Antwort noch nicht
> ausgewertet.)

Vielleicht tut sie das besser mal.

hs

Detlef Müller

unread,
Jan 1, 2017, 9:12:53 AM1/1/17
to
Am 01.01.2017 um 14:20 schrieb IV:
> "H0Iger SchuIz" schrieb im Newsbeitrag
> news:1mz65gh.1ehejlx1mciwzuN%q...@gmx.net...
>>>> Womöglich kann man in vorliegenden Thread auch anmerken, dass es
>>>> ohnehin egal sei, weil man immer noch nicht weiß, worum es
>>>> eigentlich gehen soll.
> Die Frage ist:
> Ich habe beobachtet, daß die Elementaren Funktionen (nach Ritt) (das
> hier ist nicht die Definition) einstellige einwertige Funktionen im
> Reellen oder im Komplexen sind, die Verkettungen ein- oder m e h r
> stelliger einwertiger Funktionen sind.

etwa f: R_+ --> R_+, x → x^3
ist die Verkettung f2 \circ f1 von
f1: R_+ --> (R_+)^2, x|-->(x,x^2) und
f2: (R_+)^2 --> R_+, (u,v)|--> u*v

> Elementare Funktionen, die eine elementare Funktion als Umkehrfunktion
> haben, sind aber nur solche Verkettungen e i n s t e l l i g e r
> Funktionen.

Zugleich ist aber
f: R_+ --> R_+, x → x^3
auch die Verkettung von
h1: R_+ --> R, x|--> 3 ln(x) und
h2: R --> R_+, u|--> exp(u)

> Wodurch werden alle mehrstellige Funktionen enthaltenden Verkettungen
> ausgeschlossen?

Anscheinend überhaupt nicht.
Eine Frage wäre, ob man eine Verkettung, in der Mehrstellige
und mehrwertige Funktionen auftauchen nicht auf andere
Weise als Verkettung einstelliger Funktionen darstellbar
ist.
Dies scheint mir hochgradig nicht trivial zu sein.

> Ich konnte das darauf zurückführen, daß eine mehrstellige Funktion, wenn
> sie eine Umkehrfunktion hat, eine mehrwertige Funktion als
> Umkehrfunktion hat. Eine mehrwertige Funktion, kann, wenn sie sich nicht
> durch eine einwertige Funktion ersetzen läßt, aber keine Elementare
> Funktion sein, da diese laut Definition nur einwertige Funktionen sind.
> Die Frage bleibt also:
> Die Frage bleibt also: "Wie kann man beweisen, daß eine einstellige
> zweiwertige Funktion, deren Argumente einstellige einwertige Funktionen
> ein und derselben Variablen x sind, nicht durch nur eine einwertige
> Funktion oder durch nur eine Verkettung nur einwertiger Funktionen
> ersetzt werden kann?"

Das Beispiel mit f, f1, f2, h1 und h2 dürfte ein Gegenbeispiel
zu dieser Aussage zu sein.

H0Iger SchuIz

unread,
Jan 1, 2017, 9:19:35 AM1/1/17
to
Eventuell sollte mal geklärt werden, was denn nun "ersetzen" bedeutet.
Inwiefern wird durch die Angabe einer anderen "Zusammensetzung" eine
Funktion in der gegebenen Zusammensetzung "ersetzt".

Wenn ich statt 3 = 2 + 1 nunmehr 3 = -7 + 10 schreibe, habe ich dann die
positive "2" durch eine negative Zahl "ersetzt"?

Ich befürchte, die Frage ist immer noch nicht klar. Wie soll man dann
die Antwort mathematisch exakt formulieren?

hs

Detlef Müller

unread,
Jan 1, 2017, 9:27:53 AM1/1/17
to
Ertappt :) ... natürlich müssen auch die Bildbereiche
überein stimmen - braucht man in der Vorliegenden Argumentation
zwar nicht, gehört aber zur (althergebrachten) Definition von
"Gleichheit" von Funktionen dazu.
Den Fehler hätte man bei der Sichtweise von Funktionen als
Relationen, also Teilmengen von D x W eher nicht gemacht.

IV

unread,
Jan 1, 2017, 10:10:28 AM1/1/17
to
"Detlef Müller" schrieb im Newsbeitrag
news:o4b2p4$gr8$1...@gwaiyur.mb-net.net...
> >> Ich habe beobachtet, daß die Elementaren Funktionen (nach Ritt) (das
> >> hier ist nicht die Definition) einstellige einwertige Funktionen im
> >> Reellen oder im Komplexen sind, die Verkettungen ein- oder m e h r
> >> stelliger einwertiger Funktionen sind.
> etwa f: R_+ --> R_+, x → x^3 ist die Verkettung f2 \circ f1 von
> f1: R_+ --> (R_+)^2, x|-->(x,x^2) und
> f2: (R_+)^2 --> R_+, (u,v)|--> u*v
Nach Liouville und Ritt ist jede Elementare Funktion eine (ein- oder
mehrstellige [IV]) algebraische Funktion Elementarer Funktionen (= eine
algebraische Funktion von Monomen). Die äußere Funktion der Verkettung ist
also immer eine algebraische Funktion.
f: x \mapsto x^3; f = ^3 o x
g: x \mapsto e^x; f = id o e^x
h: x \mapsto e^(x+ln(e^x)) = e^x * e^ln(e^x); h = `*`(e^,e^ln(e^)); `*` ist
eine algebraische Funktion.

> Elementare Funktionen, die eine elementare Funktion als Umkehrfunktion
> haben, sind aber nur solche Verkettungen e i n s t e l l i g e r
> Funktionen.
h: wie oben; läßt sich reduzieren auf h = e^(2x); h = id o e^ o `2*`.
h besitzt also eine Darstellung als Verkettung lediglich einstelliger ([IV])
Funktionen. Nach Ritts Satz gilt dann: Wenn h bijektiv ist, also eine
Umkehrfunktion besitzt, dann ist die Umkehrfunktion von h eine Elementare
Funktion.

>>>> Wodurch werden alle mehrstellige Funktionen enthaltenden Verkettungen
>>>> ausgeschlossen?
> Anscheinend überhaupt nicht.
Ritts Satz sagt, nur Verkettungen einstelliger Funktionen ([IV]) können eine
Elementare Funktion als Umkehrfunktion haben.

> Eine Frage wäre, ob man eine Verkettung, in der mehrstellige und
> mehrwertige Funktionen auftauchen, nicht auf andere Weise als Verkettung
> einstelliger Funktionen darstellbar ist.
> Dies scheint mir hochgradig nicht trivial zu sein.
Ritt schreibt dazu:
"It remains to develop a method for recognizing whether a given elementary
function can be reduced to the above form for F(.z). How to test fairly
simple functions will be evident from the details of our proofs."
Ich denke, daß das doch trivial ist. Denn tatsächlich gibt es im System
exp-ln-Algebraische_Funktionen nur einige wenige Kombinationen von
Komponentenfunktionen in der Verkettung, die aus einer m e h r stelligen
algebraischen Komponentenfunktion eine einstellige Funktion machen. Beispiel
siehe die Funktion h oben. (Die Kombinationen sind mir bekannt.) Leider
nennt Ritt dies ganz offensichtlichen Kombinationen nicht explizit, sondern
versteckt sie irgendwo in seinem 23-seitigen Artikel.

IV

unread,
Jan 1, 2017, 10:29:41 AM1/1/17
to
"H0Iger SchuIz" schrieb im Newsbeitrag
news:1mz6id7.rxtw1f19bp88cN%q...@gmx.net...
> Eventuell sollte mal geklärt werden, was denn nun "ersetzen" bedeutet.
> Inwiefern wird durch die Angabe einer anderen "Zusammensetzung" eine
> Funktion in der gegebenen Zusammensetzung "ersetzt".
> ...
> Ich befürchte, die Frage ist immer noch nicht klar. Wie soll man dann die
> Antwort mathematisch exakt formulieren?
(Tut mir leid, mein früheres Postimg ist irgendwie nicht gesendet worden.)
Es ist zu zeigen (bzw. zu beweisen) daß eine einstellige z w e i wertige
algebraische Funktion, deren Funktionswerte (Korrigiert!) Funktionswerte
einstelliger einwertiger algebraisch unabhängiger Funktionen ein und
derselben Variablen x sind, nicht als e i n wertige Funktion dargestellt
werden kann.
Muß man "darstellen" noch präzisieren?
Muß man den begriff "algebraisch unabhängige Funktionen" noch definieren?



Carlo XYZ

unread,
Jan 1, 2017, 2:01:36 PM1/1/17
to
Detlef Müller <lef...@arcor.de> wrote:

> Ertappt :) ... natürlich müssen auch die Bildbereiche
> überein stimmen

Caveat für die hier mitlesenden Nicht-Mathematiker: Diese sind
nicht notwendigerweise identisch mit den Wertebereichen.

> braucht man in der Vorliegenden Argumentation zwar nicht,

Bevor man nicht genau weiß, was Herr IV eigentlich meint,
halte ich das für nicht gesichert.

> gehört aber zur (althergebrachten) Definition von
> "Gleichheit" von Funktionen dazu.

Althergebracht? Weiß nicht. So weit ich die (ältere) Literatur kenne,
wird dieses verkürzende "Sei f(z) eine Funktion" leider oft verwendet.
Mir scheint eher, als könnten wir hier von der Informatik lernen, die
ja bei ungenau definierten Datenstrukturen die Krise kriegt. Zumindest
theoretisch.

> Den Fehler hätte man bei der Sichtweise von Funktionen als
> Relationen, also Teilmengen von D x W eher nicht gemacht.

Das versuche ich Herrn IV seit Wochen vergeblich nahezulegen.

IV

unread,
Jan 2, 2017, 4:29:34 PM1/2/17
to
"Carlo XYZ" schrieb im Newsbeitrag
news:carloxyz-D5E37A...@88-209-239-213.giganet.hu...
>> Wie kann man aber die Bijektivität oder Nicht-Bijektivität beweisen, wo
>> x2 = e^x1?
>> Hat jemand hier Ahnung davon? Kann mir bitte jemand helfen?
> Ist dir überhaupt klar, dass die Menge \{(x,e^x)|x\in C\} auf C\times C
> weder eine surjektive noch eine injektive Relation ist, prima facie also
> gar keine Umkehrfunktion besitzt? Vermutlich nicht, wenn dir der Begriff
> der Riemannschen Fläche nichts sagt:
Vermutlich doch, da ich ja von Umkehrfunktionen rede, und auch von lokalen
Umkehrfunktionen.
Und Riemannsche Flächen sagen mir nicht nichts, sondern bisher nur wenig.
Auch von Zweigen von Funktionen hatte ich an anderer Stelle gesprochen.

> Übungsaufgaben: Wenn f(z)=e^z also keine umkehrbare Funktion ist, wie
> verträgt sich das mit Ritts Satz, den du angeblich verstanden hast? Ist
> e^z elementar? Algebraisch? Dito log(z)? Gibt es eine Darstellung von e^z
> gemäß Ritts Satz, und wenn ja, wieso?
Über solche Fragen bin ich doch schon seit langem hinweg. All das
beantworten die Definitionen in Ritts Artikel.
e^z ist elementar, transzendent = nicht algebraisch.
Da Ritt keine Angaben über konkrete Definitionsbereiche macht, nehme ich mal
an, daß man wahlweise Gebiete nehmen kann, in denen exp bijektiv ist, und
solche, in denen exp nicht bijektiv ist.
Ritts Satz sagt nur, daß, wenn die Funktion exp eine Umkehrfunktion hat,
diese eine Elementare Funktion ist. Und ob die Funktion exp eine
Umkehrfunktion hat, hängt vom jeweiligen Definitionsbereich ab. Also kann
man Einschränkungen der Funktion betrachten.

Mein Problem ist, wie man die Bijektivität oder Nicht-Bijektivität folgender
Art von Funktionen ermitteln/zeigen kann:
A sei eine algebraische Funktion, mit z. B. A(x1,x2) = x*e^x oder A(x1,x2) =
x + e^x.
A: {(x1,x2) | (x1,x2) \in \mathbb{C}^2, x1=x, x2=e^x, x \in {\mathbb{C}}
\subseteq \mathbb{C}^2 --> Y \subseteq \mathbb{C}, (x1,x2) \mapsto y =
A(x1,x2)
Das was mir Probleme bereitet, und das ich bisher in keinem
Mathematikbuch, -skript oder -artikel gefunden habe, ist, daß die Argumente
(x1 und x2) der Funktion von derselben Variablen abhängig sind. Sie sind
also keine unabhängigen Variablen. Der Definitionsbereich ist keine Ebene
mehr, sondern eine Kurve. Unterschiedliche Mächtigkeit/Dimension von
Definitions- und Wertebereich ist also nicht mehr die Ursache für
Nicht-Bijektivität.
Wie behandelt man solche Funktionen? Wo kann ich etwas darüber lesen?


Marc Olschok

unread,
Jan 2, 2017, 10:31:56 PM1/2/17
to
IV <ivgr...@onlinehome.de> wrote:
> Hallo,
>
> f1, f2, f seien einwertige komplexwertige Funktionen (oder reellwertige
> Funktionen).
> f1, f2 sind einstellige Funktionen.
> f ist eine zweistellige Funktion.
> f^-1 ist die Umkehrfunktion von f.
> Die Argumente von f sind abhängig voneinander.
> f: (f1(z),f2(z)) \mapsto f(f1(z),f2(z)) \in \mathbb{C} (or \mathbb{R})
> f^-1: f(f1(z),f2(z)) \mapsto (f1(z),f2(z))
> Wie kann man beweisen, daß die einstellige zweiwertige Funktion f^-1 nicht
> durch nur eine einwertige Funktion oder durch nur eine Verkettung nur
> einwertiger Funktionen ersetzt werden kann?
> Kann jemand helfen?

Schwer zu sagen. Soweit ich obiges entschlüsseln konnte, verstehst Du
unter einer "n-wertigen" und "m-stelligen" Funktion eine Abbildung
von C^m nach C^n. Dementsprechend liegen also Abbildungen

f1: C --> C , f2: C --> C , f: C^2 --> C

vor, so dass f invertierbar ist, mit Umkehrfunktion f^-1: C --> C^2.
Damit kann f^-1 offensichtlich nicht mit einer Abbildung vom Typ
C --> C überein stimmen. Sofern das mit dem "[...] ersetzt wrden kann"
gemeint war, ist die Antwort als schon in die Definition einer
Abbildung eingebaut.

Allerdings bleibt unklar, wozu f1 und f2 überhaupt gebraucht werden.
Kann es sein, dass Dein f nur auf dem Bild von (f1,f2): C --> C^2
definiert sein soll?

--
Marc

Carlo XYZ

unread,
Jan 3, 2017, 12:09:04 AM1/3/17
to
"IV" <ivgr...@onlinehome.de> wrote:

> Über solche Fragen bin ich doch schon seit langem hinweg.

Ach ja? Dann gib doch mal an, ob die Beispielfunktion zu Anfang von
<carloxyz-F6F44D...@88-209-239-213.giganet.hu>
ein elementares Inverses hat und wie die Zerlegung dann aussieht.
Dito, wenn das zu schwer ist, mit F(z)=z+e^z.

> Mein Problem ist, wie man die Bijektivität oder Nicht-Bijektivität folgender
> Art von Funktionen ermitteln/zeigen kann:
> A sei eine algebraische Funktion, mit z. B. A(x1,x2) = x*e^x oder A(x1,x2) =
> x + e^x.
> A: {(x1,x2) | (x1,x2) \in \mathbb{C}^2, x1=x, x2=e^x, x \in {\mathbb{C}}
> \subseteq \mathbb{C}^2 --> Y \subseteq \mathbb{C}, (x1,x2) \mapsto y =
> A(x1,x2)

Nach Zeile 5 ist A eine zweistellige Relation, also (wie du sagst) eine
einstellige Funktion, nach Zeilen 3 und 7 aber eine (wie du sagst)
zweistellige Funktion. Das ist doch grober Unfug.

Detlef Müller

unread,
Jan 3, 2017, 7:06:35 AM1/3/17
to
Am 01.01.2017 um 16:29 schrieb IV:
> "H0Iger SchuIz" schrieb im Newsbeitrag
> news:1mz6id7.rxtw1f19bp88cN%q...@gmx.net...
[...]
> Es ist zu zeigen (bzw. zu beweisen) daß eine einstellige z w e i
> wertige algebraische Funktion, deren Funktionswerte (Korrigiert!)
> Funktionswerte einstelliger einwertiger algebraisch unabhängiger
> Funktionen ein und derselben Variablen x sind, nicht als e i n wertige
> Funktion dargestellt werden kann.

Gibt es eine eigene Definition, wann man eine Abbildung der Gestalt
f: K --> K^2, x |--> (f_1(x), f_2(x))
("zweiwertige" Funktion) algebraisch nennt?

Ich hätte jetzt vermutet, daß f einer Algebraischen
Gleichung genügt in dem Sinne, daß es ein irreduzibles Polynom
P in K[x_1,x_2,x_3] gibt, so daß P(f_1(x), f_2(x), x)
die Null-Abbildung ergibt.

PS:

Ein Blick in den zugehörigen Artikel von J. F. Ritt:

http://www.ams.org/journals/tran/1925-027-01/S0002-9947-1925-1501299-9/S0002-9947-1925-1501299-9.pdf

sagt mir zunächst, daß man hier wohl sattelfest in
Funktionentheorie sein sollte, z.B. was analytische
Fortsetzungen entlang von Pfaden betrifft, über die z.B. die
vorkommenden Logarithmen und arcus-Funktionen wohl aufzufassen
sind.

Diesen Artikel kann man durcharbeiten, wenn man entweder gerade
gut in die Thematik eingearbeitet ist oder dies anhand des
Artikels zu tun gedenkt.

Könnte schönes Thema für ein Seminar im Anschluß an eine Vorlesung
"Funktionentheorie" sein, in dem man an einem konkreten Problem
sieht, wie all das gelernte zusammenspielt, um damit wirklich zu
arbeiten und interessante Themen zu untersuchen.
Wohlgemerkt: Ein Seminar, nicht ein Vortrag.

Wenn ich mich mal wieder in die Fun-Theorie einfuchsen möchte,
wäre dieser Artikel eine gute Möglichkeit ... allerdings würde
ich da (für mich) mindestens eine Woche einplanen, um wirklich
zu verstehen, was im Detail vor sich geht (dabei würde ich dann
nach und nach das meiste aus der Fun-Theorie-Vorlesung damals
oder anderen Büchern wieder nachschlagen).

Wie gesagt: Mathematiker, die in der Funktionentheorie zu hause
sind, werden das vielleicht in wenigen Stunden durchgehen
können.

IV

unread,
Jan 3, 2017, 11:57:46 AM1/3/17
to
"Marc Olschok" schrieb im Newsbeitrag
news:o4f5v9$nro$1...@news.albasani.net...
>> f1, f2, f seien einwertige komplexwertige Funktionen (oder reellwertige
>> Funktionen).
>> f1, f2 sind einstellige Funktionen.
>> f ist eine zweistellige Funktion.
>> f^-1 ist die Umkehrfunktion von f.
>> Die Argumente von f sind abhängig voneinander.
>> f: (f1(z),f2(z)) \mapsto f(f1(z),f2(z)) \in \mathbb{C} (or \mathbb{R})
>> f^-1: f(f1(z),f2(z)) \mapsto (f1(z),f2(z))
>> Wie kann man beweisen, daß die einstellige zweiwertige Funktion f^-1
>> nicht durch nur eine einwertige Funktion oder durch nur eine Verkettung
>> nur einwertiger Funktionen ersetzt werden kann?
>> Kann jemand helfen?

> Schwer zu sagen. Soweit ich obiges entschlüsseln konnte, verstehst Du
> unter einer "n-wertigen" und "m-stelligen" Funktion eine Abbildung von C^m
> nach C^n. Dementsprechend liegen also Abbildungen f1: C --> C , f2:
> C --> C , f: C^2 --> C
vor, so dass f invertierbar ist, mit Umkehrfunktion f^-1: C --> C^2.
Nicht ganz. Der Definitionsbereich ist nicht C, sondern eine Teilmengen von
C. Ritts Satz beginnt: "If F(z) and its inverse are both elementary, there
...". Es wird also vorausgesetzt, daß die Funktion F eine Umkehrfunktion
hat, also bijektiv ist. Da, zumindest für mich nicht ersichtlich, im Artikel
keine Definitionsbereiche angegeben werden, kann man den Definitionsbereich
von F so wählen, daß F bijektiv ist und F über diesem Definitionsbereich
untersucht werden soll.

> Damit kann f^-1 offensichtlich nicht mit einer Abbildung vom Typ C --> C
> überein stimmen. Sofern das mit dem "[...] ersetzt werden kann" gemeint
> war, ist die Antwort als schon in die Definition einer Abbildung
> eingebaut.
> Allerdings bleibt unklar, wozu f1 und f2 überhaupt gebraucht werden.
> Kann es sein, dass Dein f nur auf dem Bild von (f1,f2): C --> C^2
> definiert sein soll?
Ich glaube ja. Wo denn sonst? Die Definitionsmenge D von f, D =
{(f1(x)=x,f2(x)=e^x) | x \in X \subseteq \mathbb{R}}, ist eine Kurve.
Kann die Funktion f bijektiv sein?


Ich werde in den nächsten Tagen versuchen, mein mathematisches Problem hier
noch einmal neu zu formulieren.

IV

unread,
Jan 3, 2017, 12:36:42 PM1/3/17
to
"Detlef Müller" schrieb im Newsbeitrag
news:o4g44a$c5q$1...@gwaiyur.mb-net.net...
>> Es ist zu zeigen (bzw. zu beweisen) daß eine einstellige z w e i wertige
>> algebraische Funktion, deren Funktionswerte (Korrigiert!) Funktionswerte
>> einstelliger einwertiger algebraisch unabhängiger Funktionen ein und
>> derselben Variablen x sind, nicht als e i n wertige Funktion dargestellt
>> werden kann.
> Gibt es eine eigene Definition, wann man eine Abbildung der Gestalt f:
> K --> K^2, x |--> (f_1(x), f_2(x)) ("zweiwertige" Funktion) algebraisch
> nennt?
> Ich hätte jetzt vermutet, daß f einer Algebraischen Gleichung genügt in
> dem Sinne, daß es ein irreduzibles Polynom P in K[x_1,x_2,x_3] gibt, so
> daß P(f_1(x), f_2(x), x) die Null-Abbildung ergibt.
Der Begriff "zweiwertige Funktion" ist definiert, und der Begriff
"Algebraische Funktion" auch. Eine zweiwertige algebraische Funktion ist
nach meiner Auffassung eine zweiwertige Funktion, die eine algebraische
Funktion ist, was gleichbedeutend ist mit einer algebraischen Funktion, die
eine zweiwertige Funktion ist. Deine Definition dürfte richtig sein.

> Ein Blick in den zugehörigen Artikel von J. F. Ritt:
Ritt, J. F.: Elementary functions and their inverses.
> http://www.ams.org/journals/tran/1925-027-01/S0002-9947-1925-1501299-9/S0002-9947-1925-1501299-9.pdf
> sagt mir zunächst, daß man hier wohl sattelfest in Funktionentheorie sein
> sollte, z.B. was analytische Fortsetzungen entlang von Pfaden betrifft,
> über die z. B. die vorkommenden Logarithmen und arcus-Funktionen wohl
> aufzufassen sind.
Ritts Satz beginnt folgendermaßen: "If F(z) and its inverse are both
elementary, there ...". Es wird also vorausgesetzt, daß die Funktion F eine
Umkehrfunktion hat. Man geht also davon aus, daß der Definitionsbereich
entsprechend sei. Man kann ihn vorgeben, muß es aber nicht.
Meine Beobachtung ist, daß sich Ritts Satz einfach dadurch ergibt, daß eine
mehr w e r t i g e Funktion, die eine Umkehrfunktion hat, eine mehr s t e l
l i g e Umkehrfunktion hat. Eine mehrstellige Umkehrfunktion ist aber nicht
elementar. Eine bijektive Verkettung, die eine mehrstellige (nicht
"ersetzbare"(?)) Komponentenfunktion enthält, kann also keine elementare
Funktion als Umkehrfunktion haben.

Dieser Beweis dürfte sehr einfach und kurz sein.
(Ich frage mich allerdings, warum Ritt dafür die komplexe Analysis
strapaziert und für seinen Beweis 23 Seiten benötigt.)

Carlo XYZ

unread,
Jan 3, 2017, 1:04:59 PM1/3/17
to
"IV" <ivgr...@onlinehome.de> wrote:

> Meine Beobachtung ist, daß sich Ritts Satz einfach dadurch ergibt, daß eine
> mehr w e r t i g e Funktion, die eine Umkehrfunktion hat, eine mehr s t e l
> l i g e Umkehrfunktion hat. Eine mehrstellige Umkehrfunktion ist aber nicht
> elementar. Eine bijektive Verkettung, die eine mehrstellige (nicht
> "ersetzbare"(?)) Komponentenfunktion enthält, kann also keine elementare
> Funktion als Umkehrfunktion haben.
>
> Dieser Beweis dürfte sehr einfach und kurz sein.
> (Ich frage mich allerdings, warum Ritt dafür die komplexe Analysis
> strapaziert und für seinen Beweis 23 Seiten benötigt.)

In Ritts Satz sind alle vorkommenden Funktionen, sowohl F als auch
die \phi_j, sowie (falls existent) deren Inverse, alle vom Typ C->C,
also in deiner Sprechweise einstellig, oder eher gesagt, Relationen
bzw.partielle Funktionen (als Teilmengen von C\times C). Da gibt es
überhaupt nichts "Mehrstelliges" oder "Mehrwertiges".

Du hast den Satz scheint's einfach nur nicht verstanden.

IV

unread,
Jan 3, 2017, 4:44:48 PM1/3/17
to
"Carlo XYZ" schrieb im Newsbeitrag
news:carloxyz-EEC03C...@88-209-239-213.giganet.hu...
Genau das war meine Beobachtung.

>>> Meine Beobachtung ist, daß sich Ritts Satz einfach dadurch ergibt, daß
>>> eine mehr w e r t i g e Funktion, die eine Umkehrfunktion hat, eine
>>> mehr s t e l l i g e Umkehrfunktion hat. Eine mehrstellige
>>> Umkehrfunktion ist aber nicht
>>> elementar. Eine bijektive Verkettung, die eine mehrstellige (nicht
>>> "ersetzbare"(?)) Komponentenfunktion enthält, kann also keine elementare
>>> Funktion als Umkehrfunktion haben.
> Du hast den Satz scheint's einfach nur nicht verstanden.
Du hast mich, das ist eine Tatsache, einfach nur nicht verstanden.
Ich habe beobachtet, daß die *Elementaren Funktionen* Verkettungen von exp,
ln und/oder ein- oder m e h r stelligen algebraischen (*einwertigen*)
Funktionen sind. (Die Mehrstelligkeit einer Komponentenfunktion ergibt sich,
wenn die Argumente (Ritt: Monome) einer algebraischen Komponentenfunktion
algebraisch unabhängig sind.) Daß die Umkehrfunktion einer solchen
Verkettung eine Verkettung der Umkehrfunktionen der Komponentenfunktionen
ist, folgt aus den Eigenschaften von Funktionskompositionen. Da die
Umkehrfunktion einer mehr s t e l l i g e n Komponentenfunktion eine mehr w
e r t i g e Komponentenfunktion ist, mehrwertige Funktionen aber keine
Elementaren Funktionen sind, kann die Umkehrfunktion keine Elementare
Funktion sein.

Ich werde in den nächsten Tagen versuchen, hier mein mathematisches Problem

Detlef Müller

unread,
Jan 3, 2017, 5:12:14 PM1/3/17
to
Am 03.01.2017 um 18:36 schrieb IV:
> "Detlef Müller" schrieb im Newsbeitrag
> news:o4g44a$c5q$1...@gwaiyur.mb-net.net...
[...]
>> Gibt es eine eigene Definition, wann man eine Abbildung der Gestalt f:
>> K --> K^2, x |--> (f_1(x), f_2(x)) ("zweiwertige" Funktion)
>> algebraisch nennt?
>> Ich hätte jetzt vermutet, daß f einer Algebraischen Gleichung genügt
>> in dem Sinne, daß es ein irreduzibles Polynom P in K[x_1,x_2,x_3]
>> gibt, so daß P(f_1(x), f_2(x), x) die Null-Abbildung ergibt.
> Der Begriff "zweiwertige Funktion" ist definiert, und der Begriff
> "Algebraische Funktion" auch.

In der Tat, allerdings habe ich nur Definitionen für "algebraisch"
gefunden, die inhärent benutzen, daß die Werte gegebene Funktion für
eine Variable in einem Polynom substituiert werden können.

Diese Definition funktioniert für mehrwertige bzw. vektorwertige
Funktionen nicht, da man Vektoren nicht in Polynome einsetzen
kann, also P(f(x),x)=0 keinen Sinn ergibt.

> Eine zweiwertige algebraische Funktion ist
> nach meiner Auffassung eine zweiwertige Funktion, die eine algebraische
> Funktion ist, was gleichbedeutend ist mit einer algebraischen Funktion,
> die eine zweiwertige Funktion ist. Deine Definition dürfte richtig sein.

Alternativ könnte ja auch f(x)=(f_1(x),f_2(x)) als algebraisch definiert
sein dadurch, daß die Funktionen f_1 und f_2 für sich algebraisch sind,
evtl. ist das auch gleichwertig, könnte man sich überlegen.

Genau solche Feinheiten entscheiden dann darüber, welche Sätze
aus anderen Werken man dann verwenden darf oder wo man ins Fettnäpfchen
greift.

>> Ein Blick in den zugehörigen Artikel von J. F. Ritt:
> Ritt, J. F.: Elementary functions and their inverses.
>> http://www.ams.org/journals/tran/1925-027-01/S0002-9947-1925-1501299-9/S0002-9947-1925-1501299-9.pdf
>>
>> sagt mir zunächst, daß man hier wohl sattelfest in Funktionentheorie
>> sein sollte, z.B. was analytische Fortsetzungen entlang von Pfaden
>> betrifft, über die z. B. die vorkommenden Logarithmen und
>> arcus-Funktionen wohl aufzufassen sind.
> Ritts Satz beginnt folgendermaßen: "If F(z) and its inverse are both
> elementary, there ...". Es wird also vorausgesetzt, daß die Funktion F
> eine Umkehrfunktion hat.
Zumindest lokal, ja.

> Man geht also davon aus, daß der
> Definitionsbereich entsprechend sei. Man kann ihn vorgeben, muß es aber
> nicht.

Es scheint hier um Funktionen C-->C zu gehen, die "fast überall"
definiert sind.
Dabei scheinen Überlagerungen und das "tragen" von Urbildern
entlang von Pfaden eine Rolle zu spielen.

Die ganze dahinter steckende Funktionentheorie wird im Artikel
natürlich vorausgesetzt und kommt nur an den Stellen, wo die
zugehörigen Pfade und epsilon-Umgebungen angesprochen werden
explizit zu Tage.

Zudem beschäftigt sich Ritt im Artikel sogar recht intensiv
mit Definitionsbereichen, z.B. auf p.70, wo er nach einem
Absatz, in dem es im Punkt 3. nach längerer Argumentation,
wie man gewisse Pfade (Teilmengen des Definitionsbereiches),
auf denen eine Funktion v an gewissen Stellen 0 wird (Definitionslücken
des Log), abwandelt zur Folgerung:
"Thus log(w) is analytic almost everywhere."
kommt.
Der ganze Aufwand geht darum, die betreffenden Pfade so zu
gestalten, daß der Definitionsbereich des Log (bzw. besser der
Bereich in dem Log über die Pfade definiert werden kann) eben
nicht verlassen wird!

> Meine Beobachtung ist, daß sich Ritts Satz einfach dadurch ergibt, daß
> eine mehr w e r t i g e Funktion, die eine Umkehrfunktion hat, eine
> mehr s t e l l i g e Umkehrfunktion hat. Eine mehrstellige
> Umkehrfunktion ist aber nicht elementar. Eine bijektive Verkettung, die
> eine mehrstellige (nicht "ersetzbare"(?)) Komponentenfunktion enthält,
> kann also keine elementare Funktion als Umkehrfunktion haben.

Da bei Ritt nirgends mehrstellige Funktionen auftauchen, wäre das
ein ganz anderer Ansatz. Im einleitenden
"We prove that if F(z) and its inverse are both elementary,
there exist n functions ... such that ..."
sehe ich nichts dergleichen.
Hier tauchen nur Funktionen der Sorte C --> C auf.

> Dieser Beweis dürfte sehr einfach und kurz sein.
> (Ich frage mich allerdings, warum Ritt dafür die komplexe Analysis
> strapaziert und für seinen Beweis 23 Seiten benötigt.)

Ja, das sollte einem zu denken geben.

IV

unread,
Jan 4, 2017, 12:25:27 PM1/4/17
to
"Detlef Müller" schrieb im Newsbeitrag
news:o4h7jt$et8$1...@gwaiyur.mb-net.net...
>>>> Gibt es eine eigene Definition, wann man eine Abbildung der Gestalt f:
>>>> K --> K^2, x |--> (f_1(x), f_2(x)) ("zweiwertige" Funktion) algebraisch
>>>> nennt?
>>>> Ich hätte jetzt vermutet, daß f einer Algebraischen Gleichung genügt in
>>>> dem Sinne, daß es ein irreduzibles Polynom P in K[x_1,x_2,x_3] gibt, so
>>>> daß P(f_1(x), f_2(x), x) die Null-Abbildung ergibt.
>>> Der Begriff "zweiwertige Funktion" ist definiert, und der Begriff
>>> "Algebraische Funktion" auch.
>> In der Tat, allerdings habe ich nur Definitionen für "algebraisch"
>> gefunden, die inhärent benutzen, daß die Werte gegebene Funktion für eine
>> Variable in einem Polynom substituiert werden können.
>> Diese Definition funktioniert für mehrwertige bzw. vektorwertige
>> Funktionen nicht, da man Vektoren nicht in Polynome einsetzen kann, also
>> P(f(x),x)=0 keinen Sinn ergibt.
Bei Ritt und bei Ritts Thematik geht es nirgendwo um mehrwertige
algebraische Funktionen. Solche Funktionen wären die Umkehrfunktionen von
mehrstelligen algebraischen Funktionen. Ist aber in der
Verkettungsdarstellung einer Elementaren Funktion (beides nach Ritt) eine
Komponentenfunktion mehrstellig, dann existiert nach Ritts Satz die
Umkehrfunktion der Verkettung nicht als Elementare Funktion, wird in Ritts
Satz also gar nicht betrachtet.
Von daher braucht auch nicht definiert werden, was eine mehrwertige
algebraische Funktion sei.
Ich denke folgendes.
Algebraische Funktion y, y: x \mapsto y(x): auch in Ritts Artikel so
definiert: gehorcht einer irreduziblen algebraischen Gleichung p0 +
p1(x)*y(x) + p2(x)*y(x)^2 + ... + pn(x)*y(x)^n = 0.
Algebraische Funktion y, y: x \mapsto (f1(x),f2(x)): ergibt ein
Gleichungssystem p0 + p1(x)*f1(x) + p2(x)*f1(x)^2 + ... + pn(x)*f1(x)^n = 0,
q0 + q1(x)*f2(x) + q2(x)*f2(x)^2 + ... + qn(x)*f2(x)^n = 0.
Wir hatten das hier in einem anderen Thread schon mal. Eine zweiwertige
Funktion sind praktisch zwei "parallele" Funktionen. Damals hattet Ihr
erklärt, daß solch eine zweiwertige Funktion algebraisch ist, weil ja jede
der beiden Teilfunktionen algebraisch ist.
Wo liege ich falsch?
> Alternativ könnte ja auch f(x)=(f_1(x),f_2(x)) als algebraisch definiert
> sein dadurch, daß die Funktionen f_1 und f_2 für sich algebraisch sind,
> evtl. ist das auch gleichwertig, könnte man sich überlegen.
Aha, da hast Du ja diese Interpretation auch.

> Genau solche Feinheiten entscheiden dann darüber, welche Sätze aus anderen
> Werken man dann verwenden darf oder wo man ins Fettnäpfchen greift.
Wie gesagt, solche Funktionen werden hier eigentlich gar nicht benötigt.

>> Ritts Satz beginnt folgendermaßen: "If F(z) and its inverse are both
>> elementary, there ...". Es wird also vorausgesetzt, daß die Funktion F
>> eine Umkehrfunktion hat.
> Zumindest lokal, ja.
Oder mit der entsprechenden Einschränkung des Definitionsbereiches von F.

>> Man geht also davon aus, daß der Definitionsbereich entsprechend sei. Man
>> kann ihn vorgeben, muß es aber nicht.
> Es scheint hier um Funktionen C-->C zu gehen, die "fast überall" definiert
> sind. Dabei scheinen Überlagerungen und das "Tragen" von Urbildern entlang
> von Pfaden eine Rolle zu spielen.
Wo in Ritts Artikel steht, daß es bei F um eine Funktion C --> C geht?
Ich denke mal, es geht um F: D \subseteq C --> Z \subseteq C. Das wird an
mehreren Stellen deutlich. Ritt schränkt sowohl die Monome (= äußere
Funktion ist exp oder ln) als auch die algebraischen Funktionen auf Gebiete,
auf denen sie definiert und analytisch sind, ein. Und damit trifft das dann
auch für D zu.
Wo liege ich falsch?
Wären die algebraischen Funktionen, die Ritt meint, über ganz C definiert,
dann dürften ja nur die linearen Funktionen gemeint sein. Weil doch nur
diese bijektiv sind. Zu den Elementaren Funktionen, auch in Ritts Definition
gleich zu Anfang des Artikels, gehören aber natürlich auch die nichtlinearen
algebraischen Funktionen, und solche Funktionen, die in ihrer
Verkettungsdarstellung solche nichtlinearen algebraischen
Komponentenfunktionen haben.
Wo liege ich falsch?

>> Meine Beobachtung ist, daß sich Ritts Satz einfach dadurch ergibt, daß >
>> eine mehr w e r t i g e Funktion, die eine Umkehrfunktion hat, eine mehr
>> s t e l l i g e Umkehrfunktion hat. Eine mehrstellige Umkehrfunktion ist
>> aber nicht elementar. Eine bijektive Verkettung, die eine mehrstellige
>> (nicht "ersetzbare"(?)) Komponentenfunktion enthält, kann also keine
>> elementare Funktion als Umkehrfunktion haben.
> Da bei Ritt nirgends mehrstellige Funktionen auftauchen, wäre das ein ganz
> anderer Ansatz. Im einleitenden "We prove that if F(z) and its inverse are
> both elementary, there exist n functions ... such that ..." sehe ich
> nichts dergleichen.
Ja, es ist ein ganz anderer Ansatz. Ich denke mal, es ist ein algebraischer
Ansatz.

> Im einleitenden "We prove that if F(z) and its inverse are both
> elementary, there exist n functions ... such that ..." ...
> Hier tauchen nur Funktionen der Sorte C --> C auf.
Auch D \subseteq C --> Z \subseteq C ist zugelassen, oder? Siehe oben.

>> Dieser Beweis dürfte sehr einfach und kurz sein. (Ich frage mich
>> allerdings, warum Ritt dafür die komplexe Analysis strapaziert und für
>> seinen Beweis 23 Seiten benötigt.)
> Ja, das sollte einem zu denken geben.
Dieser Beweis dürfte sehr einfach und kurz sein.
Das sollte einem zu denken geben.

IV

unread,
Jan 5, 2017, 1:26:26 PM1/5/17
to
"Detlef Müller" schrieb im Newsbeitrag
news:o4b2p4$gr8$1...@gwaiyur.mb-net.net...
> Eine Frage wäre, ob eine Verkettung, in der mehrstellige und mehrwertige
> Funktionen auftauchen, nicht auf andere Weise als Verkettung einstelliger
> Funktionen darstellbar ist.
> Dies scheint mir hochgradig nicht-trivial zu sein.
Das erste Arbeitspaket soll ja erstmal nur der Beweis des Rittschen Satzes
mit algebraischen Mitteln sein. Ich denke, das könnte doch wertvoll sein.
Der Satz von Ritt liefert nämlich bereits sofort eine Aussage für alle
Elementaren Funktionen, für die eine "lineare" Verkettung bekannt ist: Ist
die Elementare Funktion F umkehrbar und ist F als einstellige (= lineare)
Verkettung darstellbar, dann ist F elementar umkehrbar.

In einigen Tage werde ich versuchen, hier mein mathematisches Problem noch
einmal neu zu formulieren.


H0Iger SchuIz

unread,
Jan 6, 2017, 6:39:12 AM1/6/17
to
IV <ivgr...@onlinehome.de> wrote:

> "Detlef Müller" schrieb im Newsbeitrag
> news:o4b2p4$gr8$1...@gwaiyur.mb-net.net...
> > Eine Frage wäre, ob eine Verkettung, in der mehrstellige und mehrwertige
> > Funktionen auftauchen, nicht auf andere Weise als Verkettung einstelliger
> > Funktionen darstellbar ist.
> > Dies scheint mir hochgradig nicht-trivial zu sein.
> Das erste Arbeitspaket soll ja erstmal nur der Beweis des Rittschen Satzes
> mit algebraischen Mitteln sein. Ich denke, das könnte doch wertvoll sein.

Inwiefern? Welche neune Erkenntnisse leifert ein weiterer Beweis?

> Der Satz von Ritt liefert nämlich bereits sofort eine Aussage für alle
> Elementaren Funktionen, für die eine "lineare" Verkettung bekannt ist:

Die liefert er bereits mit dem von Ritt daselbst vorgelegten Beweis.

hs

IV

unread,
Jan 6, 2017, 11:28:10 AM1/6/17
to
"H0Iger SchuIz" schrieb im Newsbeitrag
news:1mzfhwq.19z1qj218ysgpeN%q...@gmx.net...
>> Das erste Arbeitspaket soll ja erstmal nur der Beweis des Rittschen
>> Satzes mit algebraischen Mitteln sein. Ich denke, das könnte doch
>> wertvoll sein. Der Satz von Ritt liefert nämlich bereits sofort eine
>> Aussage für alle Elementaren Funktionen, für die eine "lineare"
>> Verkettung bekannt ist: Ist die Elementare Funktion F umkehrbar und ist
>> F als einstellige (= lineare) Verkettung darstellbar, dann ist F
>> elementar umkehrbar.
> Inwiefern? Welche neuen Erkenntnisse liefert ein weiterer Beweis?
1.) Ein anderer Beweis mit anderen Mittel wäre schon mal ein Gewinn für die
Mathematik.
2.) Dieser Beweis könnte darüber hinaus recht kurz und leicht verständlich
sein. Ritt benötigt für seinen analytisch geführten Beweis einen ganzen
Aufsatz von 23 Seiten! Und allein deshalb schon scheinen Ritts Beweis und
Satz nicht gerade bekannt zu sein, obwohl die Frage, welche Funktionen eine
Elementare Funktion als Umkehrfunktion haben, immer wieder gestellt wird.
3.) Die oben genannte sofort aus Ritts Satz folgende Aussage ist es wert,
allgemein bekanntgemacht zu werden.
4.) Da der neue Beweis nicht mehr wie bei Liouville und Ritt auf dem
strukturellen Aufbau der Elementaren Funktionen (Grad einer Elementaren
Funktion) basiert, ist ein verallgemeinerter Satz für die sehr allgemeine
Funktionenklasse der standardfunktion-basierten Funktionen möglich.
Siehe den Thread "Zusammenarbeit gesucht: Umkehrfunktionen und allgemeine
Lösungsmethoden für Gleichungen in endlichen Ausdrücken".

H0Iger SchuIz

unread,
Jan 6, 2017, 11:40:59 AM1/6/17
to
IV <ivgr...@onlinehome.de> wrote:

> "H0Iger SchuIz" schrieb im Newsbeitrag
> news:1mzfhwq.19z1qj218ysgpeN%q...@gmx.net...
> >> Das erste Arbeitspaket soll ja erstmal nur der Beweis des Rittschen
> >> Satzes mit algebraischen Mitteln sein. Ich denke, das könnte doch
> >> wertvoll sein. Der Satz von Ritt liefert nämlich bereits sofort eine
> >> Aussage für alle Elementaren Funktionen, für die eine "lineare"
> >> Verkettung bekannt ist: Ist die Elementare Funktion F umkehrbar und ist
> >> F als einstellige (= lineare) Verkettung darstellbar, dann ist F
> >> elementar umkehrbar.
> > Inwiefern? Welche neuen Erkenntnisse liefert ein weiterer Beweis?
> 1.) Ein anderer Beweis mit anderen Mittel wäre schon mal ein Gewinn für die
> Mathematik.

Einen neuen Satz zu beweisen, wäre ein größerer Gewinn.

> 2.) Dieser Beweis könnte darüber hinaus recht kurz und leicht verständlich
> sein.

Könnte. Weiß man nicht. Kann man beurteilen, falls man tatsächlich einen
anderen Beweis gefunden haben sollte.

> Ritt benötigt für seinen analytisch geführten Beweis einen ganzen
> Aufsatz von 23 Seiten!

Ja? Und? Du meinst das ist lang? Und weil das (zu) lang ist, muss es
noch einen kürzeren Beweis geben? Falscher Ansatz.

> Und allein deshalb schon scheinen Ritts Beweis und
> Satz nicht gerade bekannt zu sein,

Ganz im Gegensatz zu z.B. Fermats letztem Satz. Der Beweis war noch mal
in etwa wie kurz? Der passt ja noch nicht mal auf den Rand eines Buches.

> obwohl die Frage, welche Funktionen eine
> Elementare Funktion als Umkehrfunktion haben, immer wieder gestellt wird.

Ist das so? Zu diesem Überblick über den Alltag mathematischer Forschung
bist du in etwa wie gelangt?

> 3.) Die oben genannte sofort aus Ritts Satz folgende Aussage ist es wert,
> allgemein bekanntgemacht zu werden.

Muss man das denn, wenn die Frage "immer wieder gestellz wird"? Du
meinst, die Antwort wird deshalb nicht gefunden, weil der Beweis "so
lang" ist?

> 4.) Da der neue Beweis nicht mehr wie bei Liouville und Ritt auf dem
> strukturellen Aufbau der Elementaren Funktionen (Grad einer Elementaren
> Funktion) basiert, ist ein verallgemeinerter Satz für die sehr allgemeine
> Funktionenklasse der standardfunktion-basierten Funktionen möglich.

Ja, kann sein. Kann amn sich anschauen, wenn eine Verallgemeinerung
formuliert und bewiesen wurde.

> Siehe den Thread "Zusammenarbeit gesucht: Umkehrfunktionen und allgemeine
> Lösungsmethoden für Gleichungen in endlichen Ausdrücken".

Wann ist mit der Veröffentlichung der Ergebnisse zu rechnen?

hs

IV

unread,
Jan 6, 2017, 12:11:27 PM1/6/17
to
"H0Iger SchuIz" schrieb im Newsbeitrag
news:1mzfxzc.89ilty9kz0wqN%q...@gmx.net...
>>>> Das erste Arbeitspaket soll ja erstmal nur der Beweis des Rittschen
>>>> Satzes mit algebraischen Mitteln sein. Ich denke, das könnte doch
>>>> wertvoll sein. Der Satz von Ritt liefert nämlich bereits sofort eine
>>>> Aussage für alle Elementaren Funktionen, für die eine "lineare"
>>>> Verkettung bekannt ist: Ist die Elementare Funktion F umkehrbar und
>>>> ist F als einstellige (= lineare) Verkettung darstellbar, dann ist F
>>>> elementar umkehrbar.
>>> Inwiefern? Welche neuen Erkenntnisse liefert ein weiterer Beweis?
>> 1.) Ein anderer Beweis mit anderen Mittel wäre schon mal ein Gewinn für
>> die Mathematik.
> Einen neuen Satz zu beweisen, wäre ein größerer Gewinn.
Ein anderer Beweis mit anderen Mittel wäre schon mal ein Gewinn für die
Mathematik.

>> Ritt benötigt für seinen analytisch geführten Beweis einen ganzen Aufsatz
>> von 23 Seiten!
> Ja? Und? Du meinst das ist lang?
Falsche Schlußfolgerung.
> Und weil das (zu) lang ist, muss es noch einen kürzeren Beweis geben?
> Falscher Ansatz.
Falsche Schlußfolgerung.

>> Und allein deshalb schon scheinen Ritts Beweis und Satz nicht gerade
>> bekannt zu sein,
> Ganz im Gegensatz zu z. B. Fermats letztem Satz. Der Beweis war noch mal
> in etwa wie kurz?
Ein einfacherer Beweis könnte dem Satz von Ritt zu weiterer Bekanntheit
verhelfen.

>> obwohl die Frage, welche Funktionen eine Elementare Funktion als
>> Umkehrfunktion haben, immer wieder gestellt wird.
> Ist das so? Zu diesem Überblick über den Alltag mathematischer Forschung
> bist du in etwa wie gelangt?
Wieso Forschung?
Im Naturwissenschaftsstudium haben wir diese Frage in der
Mathematikausbildung und in unserem eigenen Fach gestellt, aber kein
Mathematiker konnte darauf antworten.
Siehe Google: "elementary inverse", "elementary solution", "closed-form
inverse", "closed-form solution" und ähnliche Begriffe in Deutsch und
Englisch.

>> 3.) Die oben genannte sofort aus Ritts Satz folgende Aussage ist es wert,
>> allgemein bekanntgemacht zu werden.
> Muss man das denn, wenn die Frage "immer wieder gestellt wird"?
Ja, weil es nie eine Antwort gibt, obwohl doch der Satz von Ritt die Antwort
liefert.

> Du meinst, die Antwort wird deshalb nicht gefunden, weil der Beweis "so
> lang" ist?
Ein einfacherer Beweis könnte dem Satz von Ritt zu weiterer Bekanntheit
verhelfen.

> 4.) Da der neue Beweis nicht mehr wie bei Liouville und Ritt auf dem
> strukturellen Aufbau der Elementaren Funktionen (Grad einer Elementaren
> Funktion) basiert, ist ein verallgemeinerter Satz für die sehr allgemeine
> Funktionenklasse der standardfunktion-basierten Funktionen möglich.

>> Siehe den Thread "Zusammenarbeit gesucht: Umkehrfunktionen und allgemeine
>> Lösungsmethoden für Gleichungen in endlichen Ausdrücken".
> Wann ist mit der Veröffentlichung der Ergebnisse zu rechnen?
Die einzige Schwierigkeit, die ich sehe, ist die Lösung des Problems aus dem
Betreff hier.
Detlef Müller hat es so formuliert: "Eine Frage wäre, ob man eine
Verkettung, in der mehrstellige und mehrwertige Funktionen auftauchen nicht
auf andere Weise als Verkettung einstelliger Funktionen darstellbar ist.
Dies scheint mir hochgradig nicht trivial zu sein."
Wenn das gelöst ist, ist der Rest recht einfach.
Das Formulieren eines Artikels dauert dann erfahrungsgemäß natürlich auch
noch länger.
Wenn sich ein interessierter Mathematiker findet, dann geht's
selbstverständlich wesentlich schneller.


In den nächsten Tagen werde ich versuchen, mein mathematisches Problem hier

IV

unread,
Jan 6, 2017, 12:25:35 PM1/6/17
to
"Detlef Müller" schrieb im Newsbeitrag
news:o4b2p4$gr8$1...@gwaiyur.mb-net.net...
>>> Die Frage ist:
>>> Ich habe beobachtet, daß die Elementaren Funktionen (nach Ritt) (das
>>> hier ist nicht die Definition) einstellige einwertige Funktionen im
>>> Reellen oder im Komplexen sind, die Verkettungen ein- oder m e h r
>>> stelliger einwertiger Funktionen sind. Elementare Funktionen, die eine
>>> elementare Funktion als Umkehrfunktion haben, sind aber nur solche
>>> Verkettungen e i n s t e l l i g e r Funktionen.
>>> Wodurch werden alle mehrstellige Funktionen enthaltenden Verkettungen
>>> ausgeschlossen?
> Anscheinend überhaupt nicht.
> Eine Frage wäre, ob man eine Verkettung, in der mehrstellige und
> mehrwertige Funktionen auftauchen nicht auf andere Weise als Verkettung
> einstelliger Funktionen darstellbar ist.
> Dies scheint mir hochgradig nicht trivial zu sein.
Es muß Elementare Funktionen geben, die nicht "linearisierbar" sind, denn
sonst würden doch für jede umkehrbare Elementare Funktion ("lineare")
Darstellungen wie in Ritts Satz existieren. Und dann würde Ritts Satz
besagen, daß jede umkehrbare Elementare Funktion eine Elementare Funktion
als Umkehrfunktion hat. Das aber ist falsch.
Es wäre doch interessant, zu sehen, an welchen Stellen Ritt in seinem Beweis
die mehrstelligen nicht "linearisierbaren" Funktionen ausschließt. Er
scheint ja das "hochgradig nichttriviale" Problem gelöst zu haben.
Interessant wäre dann natürlich auch, ob man seine Argumentation auf andere
Funktionenklassen anwenden kann.


H0Iger SchuIz

unread,
Jan 6, 2017, 12:55:00 PM1/6/17
to
IV <ivgr...@onlinehome.de> wrote:

> "H0Iger SchuIz" schrieb im Newsbeitrag
> news:1mzfxzc.89ilty9kz0wqN%q...@gmx.net...
> >>>> Das erste Arbeitspaket soll ja erstmal nur der Beweis des Rittschen
> >>>> Satzes mit algebraischen Mitteln sein. Ich denke, das könnte doch
> >>>> wertvoll sein. Der Satz von Ritt liefert nämlich bereits sofort eine
> >>>> Aussage für alle Elementaren Funktionen, für die eine "lineare"
> >>>> Verkettung bekannt ist: Ist die Elementare Funktion F umkehrbar und
> >>>> ist F als einstellige (= lineare) Verkettung darstellbar, dann ist F
> >>>> elementar umkehrbar.
> >>> Inwiefern? Welche neuen Erkenntnisse liefert ein weiterer Beweis?
> >> 1.) Ein anderer Beweis mit anderen Mittel wäre schon mal ein Gewinn für
> >> die Mathematik.
> > Einen neuen Satz zu beweisen, wäre ein größerer Gewinn.
> Ein anderer Beweis mit anderen Mittel wäre schon mal ein Gewinn für die
> Mathematik.

Mir ist nicht klar, worin der "Gewinn" für die Mathematik besteht. Muss
ja auch nicht.

> >> Ritt benötigt für seinen analytisch geführten Beweis einen ganzen Aufsatz
> >> von 23 Seiten!
> > Ja? Und? Du meinst das ist lang?
> Falsche Schlußfolgerung.

Das war eine Frage, keine Schlussfolgerung. Mir ist nicht klar, welche
wesentliche Information mit der Nennung der Seitenzahl einhergeht.

> > Und weil das (zu) lang ist, muss es noch einen kürzeren Beweis geben?
> > Falscher Ansatz.
> Falsche Schlußfolgerung.

Auch hier war zunächst eine Frage gestellt.

> >> Und allein deshalb schon scheinen Ritts Beweis und Satz nicht gerade
> >> bekannt zu sein,
> > Ganz im Gegensatz zu z. B. Fermats letztem Satz. Der Beweis war noch mal
> > in etwa wie kurz?
> Ein einfacherer Beweis könnte dem Satz von Ritt zu weiterer Bekanntheit
> verhelfen.

Wie möcht' das gehen? Du meinst, jemand, der sich für diese
Fragestellung interessiert, findet den Satz von Ritt mit höherer
Wahrscheinlichkeit, wenn der Beweis kürzer ist?

> >> obwohl die Frage, welche Funktionen eine Elementare Funktion als
> >> Umkehrfunktion haben, immer wieder gestellt wird.
> > Ist das so? Zu diesem Überblick über den Alltag mathematischer Forschung
> > bist du in etwa wie gelangt?
> Wieso Forschung?

Ach, darum geht's nicht?

> Im Naturwissenschaftsstudium

Es soll ja auch naturwissenschaftliche Forschung geben. Aber das nur
nebenbei.

> haben wir diese Frage in der
> Mathematikausbildung und in unserem eigenen Fach gestellt, aber kein
> Mathematiker konnte darauf antworten.
> Siehe Google: "elementary inverse", "elementary solution", "closed-form
> inverse", "closed-form solution" und ähnliche Begriffe in Deutsch und
> Englisch.

Und dabei ging es um die Beweislänge oder um Rechentechniken? Um einen
Satz anwenden zu können, muss man den Beweis gar nicht Detail
durcharbeiten. Wenn ich mich recht entsinne, machen das
Naturwissenschaftler auch gar nicht immer. Manchmal prpfen sie noch
nicht mal, ob die Voraussetzung des Satzes erfüllt sind -- das sollte
man dann aber schon.

> >> 3.) Die oben genannte sofort aus Ritts Satz folgende Aussage ist es wert,
> >> allgemein bekanntgemacht zu werden.
> > Muss man das denn, wenn die Frage "immer wieder gestellt wird"?
> Ja, weil es nie eine Antwort gibt, obwohl doch der Satz von Ritt die Antwort
> liefert.

Verstehe ich nicht. Wiese gibt es denn dann keine Antwort?

> > Du meinst, die Antwort wird deshalb nicht gefunden, weil der Beweis "so
> > lang" ist?
> Ein einfacherer Beweis könnte dem Satz von Ritt zu weiterer Bekanntheit
> verhelfen.

Kann sein, muss nicht. Warum das so sein sollte, verstehe ich nicht.

> > 4.) Da der neue Beweis nicht mehr wie bei Liouville und Ritt auf dem
> > strukturellen Aufbau der Elementaren Funktionen (Grad einer Elementaren
> > Funktion) basiert, ist ein verallgemeinerter Satz für die sehr allgemeine
> > Funktionenklasse der standardfunktion-basierten Funktionen möglich.
>
> >> Siehe den Thread "Zusammenarbeit gesucht: Umkehrfunktionen und allgemeine
> >> Lösungsmethoden für Gleichungen in endlichen Ausdrücken".
> > Wann ist mit der Veröffentlichung der Ergebnisse zu rechnen?
> Die einzige Schwierigkeit, die ich sehe, ist die Lösung des Problems aus dem
> Betreff hier.
> Detlef Müller hat es so formuliert: "Eine Frage wäre, ob man eine
> Verkettung, in der mehrstellige und mehrwertige Funktionen auftauchen nicht
> auf andere Weise als Verkettung einstelliger Funktionen darstellbar ist.
> Dies scheint mir hochgradig nicht trivial zu sein."
> Wenn das gelöst ist, ist der Rest recht einfach.
> Das Formulieren eines Artikels dauert dann erfahrungsgemäß natürlich auch
> noch länger.

Also bisauf die Lösung der Probleme nd des Schreiben des Artikels bis du
schon fertig.

> Wenn sich ein interessierter Mathematiker findet,

Die stehen noch nicht Schlange, wenn sie sich an einem Gewinn für die
Mathematik beteiligen können?

Es spricht ja nichts dagegen, sich mit Ritts Satz zu beschäftigen, nach
neuen Beweisen zu suchen oder gar welche zu finden. Wem so was Spaß
macht. Dass es sich aber um eine in der zeitgenössischen Mathematik
sonderlich wichtige Fragestellung handele, kann ich nicht sehen.
Insofern kann es tatsächlich so sein, dass sich gerade niemand für diese
Fragestellung interessiert und auch niemand Interesse an einer
Zusammenarbeit hat.

Die Propaganda, wie wichtig dieser neue Beweis wäre, wird dann leider
auch etwas übertrieben.

hs

IV

unread,
Jan 6, 2017, 2:02:28 PM1/6/17
to
"H0Iger SchuIz" schrieb im Newsbeitrag
news:1mzg10p.143fttso3uvysN%q...@gmx.net...
>>>> 3.) Die oben genannte sofort aus Ritts Satz folgende Aussage ist es
>>>> wert, allgemein bekanntgemacht zu werden.
>>> Muss man das denn, wenn die Frage "immer wieder gestellt wird"?
>> Ja, weil es nie eine Antwort gibt, obwohl doch der Satz von Ritt die
>> Antwort liefert.
> Verstehe ich nicht. Wieso gibt es denn dann keine Antwort?
Man bekommt von den Mathematikern keine Antworten - oder nur sehr
unvollständige Antworten. Ich nehme an, weil die Antwort, die ja Ritt 1924
mit seinem Satz gefunden hat, weitgehend unbekannt ist.
Siehe Google: "elementary inverse", "elementary solution", "closed-form
inverse", "closed-form solution" und ähnliche Begriffe in Deutsch und
Englisch.

>>>> Siehe den Thread "Zusammenarbeit gesucht: Umkehrfunktionen und
>>>> allgemeine Lösungsmethoden für Gleichungen in endlichen Ausdrücken".
>>> Wann ist mit der Veröffentlichung der Ergebnisse zu rechnen?
>> Die einzige Schwierigkeit, die ich sehe, ist die Lösung des Problems aus
>> dem Betreff hier. Wenn das gelöst ist, ist der Rest recht einfach. Das
>> Formulieren eines Artikels dauert dann erfahrungsgemäß natürlich auch
>> noch länger.
> Also bis auf die Lösung der Probleme und das Schreiben des Artikels bist
> du schon fertig.
Das ist mathematisch korrekt. Überlege selbst, welche Stufen es in einem
Projekt gibt. Und überlege selbst, welche Schritte bis hierhin bereits
bearbeitet werden mußten bzw. konnten.

> Es spricht ja nichts dagegen, sich mit Ritts Satz zu beschäftigen, nach
> neuen Beweisen zu suchen oder gar welche zu finden. Wem so was Spaß macht.
> Dass es sich aber um eine in der zeitgenössischen Mathematik sonderlich
> wichtige Fragestellung handele, kann ich nicht sehen.
Das hat ja niemand behauptet. Das Arbeitsergebnis des Projekts wären aber
"Nice to have"-Ergebnisse.

> Insofern kann es tatsächlich so sein, dass sich gerade niemand für diese
> Fragestellung interessiert
Endlich mal eine zutreffende Schlußfolgerung.

> Insofern kann es tatsächlich so sein, dass (...) auch niemand Interesse an
> einer Zusammenarbeit hat.
Na, die Mathematiker, auch Ihr, werden gerade damit besschäftigt sein,
andere Themen der Mathematik weiterzuentwickeln.

> Die Propaganda, wie wichtig dieser neue Beweis wäre, wird dann leider auch
> etwas übertrieben.
Seh' ich nicht so.

IV

unread,
Jan 6, 2017, 7:05:34 PM1/6/17
to
"Detlef Müller" schrieb im Newsbeitrag
news:o4g44a$c5q$1...@gwaiyur.mb-net.net...
> Gibt es eine eigene Definition, wann man eine Abbildung der Gestalt f:
> K --> K^2, x |--> (f_1(x), f_2(x)) ("zweiwertige" Funktion) algebraisch
> nennt?
Salazar: The representability hierarchy and Hilbert’s 13th problem:
siehe dort Definition 1.2: "An algebraic function of m variables and n
values (over k) is a partial map of the form ..."
Arnold: TOPOLOGICAL INVARIANTS FUNCTIONS. II
siehe dort "Many-valued Mappings. A mapping f ..."


IV

unread,
Jan 6, 2017, 7:22:34 PM1/6/17
to
"IV" schrieb im Newsbeitrag news:o4pbcd$c1v$1...@news.albasani.net...
Korrektur:
Arnold: Topological invariants of algebraic functions. II
http://link.springer.com/chapter/10.1007%2F978-3-642-31031-7_24

IV

unread,
Jan 6, 2017, 9:20:04 PM1/6/17
to
"Detlef Müller" schrieb im Newsbeitrag
news:o4b2p4$gr8$1...@gwaiyur.mb-net.net...
> Eine Frage wäre, ob man eine Verkettung, in der mehrstellige und
> mehrwertige Funktionen auftauchen nicht auf andere Weise als Verkettung
> einstelliger Funktionen darstellbar ist.
> Dies scheint mir hochgradig nicht trivial zu sein.
Siehe Arnold: Topological invariants of algebraic functions. II
http://phdtree.org/pdf/?q=Topological+invariants+of+algebraic+functions
Könnte Arnolds "THEOREM ON NON-REPRESENTABILITY" bei der Beantwortung
helfen? Es trifft aber nur für ganze Funktionen zu.
Außerdem sind in meinem Problem die Argumente zwar algebraisch unabhängig
voneinander, aber doch voneinander abhängig.

Mein Problem ist ja: F(z) = A(f1(z),f2(z)), mit A, F, f1, f2, z im
Komplexen, A eine algebraische Funktion.
Was ist der Definitionsbereich von A? Im Reellen verläuft der Graph der
Funktion A über der ebenen Kurve e^z.
Kann A bijektiv sein, also eine Umkehrfunktion haben?
Ist die Umkehrfunktion von A eine zweiwertige algebraische Funktion?
Kann eine zweiwertige algebraische Funktion überhaupt als endliche
Verkettung nur einstelliger Funktionen dargestellt werden (Pendant zum
Non-Representability Theorem von Arnold(?))?


IV

unread,
Jan 6, 2017, 9:24:28 PM1/6/17
to
"IV" schrieb im Newsbeitrag news:o4pj8j$p26$1...@news.albasani.net...
> Mein Problem ist ja: F(z) = A(f1(z),f2(z)), mit A, F, f1, f2, z im
> Komplexen, A eine algebraische Funktion.
> Was ist der Definitionsbereich von A? Im Reellen verläuft der Graph der
> Funktion A über der ebenen Kurve e^z.
"Im Reellen verläuft der Graph der Funktion A über der ebenen Kurve e^z."
betrifft das Beispiel F(z) = A(z,e^z), also z. B. x*e^z oder z + e^z.

IV

unread,
Jan 7, 2017, 8:30:42 AM1/7/17
to
"Torn Rumero DeBrak" schrieb im Newsbeitrag
news:o49ho4$10lf$1...@gioia.aioe.org...
>>>> Zunächst muss es eine Funktion überhaupt geben, damit man sie ersetzen
>>>> kann. Man sollte sich also zunächst Gedanken über die Bijektivität
>>>> machen.
>>> Deinem Aufschrei folgend sehe ich folgende Fälle für die Funktionen A.
>> d) x1 \in I1, x2 \in I2; I1, I2: Intervall \subseteq \mathbb{R}; A:
>> {(x1,x2) \in I1 x I2 | x2 = e^x1} --> \mathbb{R}, (x1,x2) \mapsto x1*x2
>> Ohne die Bedingung x2 = e^x1 könnte man den Brouwerschen Satz von der
>> Invarianz der Dimension heranziehen und damit beweisen, daß die einzelnen
>> Funktionen A nicht bijektiv sind.
>> Wie kann man aber die Bijektivität oder Nicht-Bijektivität beweisen, wo
>> x2 = e^x1?
>> Hat jemand hier Ahnung davon? Kann mir bitte jemand helfen?
> Ja wie beweist du denn sonst die Bijektivität? Du mußt Surjektivität und
> Injektivität zeigen, oder, falls du die nicht-Bijektivität zeigen willst,
> mußt du ein Beispiel finden.
> Nehme einmal im Fall d) ..
> Nun ist die Lambertsche Funktion aber in [-1/e, 0) nicht injektiv, d.h.
> falls y1 in diesem Intervall liegt, gibt es zwei Werte x1,1 und x1,2 mit
> x1,1 * e^x1,1 = y1 und x1,2 * e^x1,2 = y1.
> Damit erhälst du auch 2 Paare (x1,1 , x2,1) und (x1,2 , x2,2) mit der
> gesuchten Eigenschaft und damit ist die Umkehrfunktion nicht bijektiv.
Gerade deshalb betrachte ich die Funktionen nicht über ganz \mathbb{C},
sondern über Einschränkungen, z. B. wie im Fall d) über einem Intervall in
\mathbb{R} oder \mathbb{C}.
Ein einfacheres Beispiel wäre die Funktion x\mapsto x2 = x+e^x, die ist über
ganz \mathbb{R} bijektiv.

IV

unread,
Jan 7, 2017, 8:36:35 AM1/7/17
to
"Stefan Ram" schrieb im Newsbeitrag
news:Variablen-20...@ram.dialup.fu-berlin.de...
>> Das was mir Probleme bereitet, und das ich bisher in keinem
>> Mathematikbuch, -skript oder -artikel gefunden habe, ist, daß die
>> Argumente (x1 und x2) der Funktion von derselben Variablen abhängig sind.
>> Sie sind also keine unabhängigen Variablen.
> Falls Du eine Funktion der Gestalt
> G: R --> R, x :--> F( f( x ), g( x ))
> meinst, so ist dies eine Funktion R --> R, für die es im allgemeinen keine
> besonderen Regeln gibt.
> Wenn beispielsweise f( x )= x² und g( x )= x³ und F( x, y )= x + y, dann
> wäre G( x )= x² + x³. Hier braucht man keine besonderen Regeln, soweit ich
> sehe.
Aber meine Idee war doch, die Funktion (f(x),g(x)) \mapsto F(f(x),g(x)) zu
betrachten. Wo ist etwas über solcherart Funktionen/Kompositionen
beschrieben? Haben diese vielleicht einen bestimmten Namen?


IV

unread,
Jan 7, 2017, 8:45:25 AM1/7/17
to
"Carlo XYZ" schrieb im Newsbeitrag
news:carloxyz-798BFF...@88-209-239-213.giganet.hu...
>> Mein Problem ist, wie man die Bijektivität oder Nicht-Bijektivität
>> folgender Art von Funktionen ermitteln/zeigen kann:
>> A sei eine algebraische Funktion, mit z. B. A(x1,x2) = x*e^x oder
>> A(x1,x2) = x + e^x.
>> A: {(x1,x2) | (x1,x2) \in \mathbb{C}^2, x1=x, x2=e^x, x \in {\mathbb{C}}
>> \subseteq \mathbb{C}^2 --> Y \subseteq \mathbb{C}, (x1,x2) \mapsto y =
>> A(x1,x2)
> Nach Zeile 5 ist A eine zweistellige Relation, also (wie du sagst) eine
> einstellige Funktion, nach Zeilen 3 und 7 aber eine (wie du sagst)
> zweistellige Funktion. Das ist doch grober Unfug.
Und wie ist es richtig?
Kann man etwas über die Anzahl der Argumente der Funktion A sagen?
Was ist der Definitionsbereich der Funktion A?
Kann A bijektiv sein?
Kann A über \mathbb{R} bijektiv sein?
Wo kann ich etwas zu solchen (mehrstelligen(?)) Funktionskompositionen
finden?
Wird sowas in der Algebraischen Geometrie behandelt?

IV

unread,
Jan 7, 2017, 8:58:14 AM1/7/17
to
"Carlo XYZ" schrieb im Newsbeitrag
news:carloxyz-393077...@88-209-239-213.giganet.hu...
>> Den Fehler hätte man bei der Sichtweise von Funktionen als Relationen,
>> also Teilmengen von D x W eher nicht gemacht.
> Das versuche ich Herrn IV seit Wochen vergeblich nahezulegen.
Das habe ich noch nicht bemerkt. Ihr wollt, daß ich immer konkrete
Definitionsmengen und Zielmengen angebe. Wie wäre denn die
Relationenschreibweise meiner Funktionen F und A:
F: X \subseteq C --> Z \subseteq C, x \mapsto F(x) = A(f1(x),f2(x)). f1, f2:
C --> C; A: eine algebraische Funktion


0 new messages